You are on page 1of 356

Option Markets 232D

1. Introduction
Daniel Andrei

Fall 2012

1 / 67

My Background

MScF 2006, PhD 2012. Lausanne, Switzerland

Since July 2012: assistant professor of finance at UCLA Anderson

I conduct research in the area of theoretical asset pricing, with a


special focus on the role of information in financial markets

More information about me at www.danielandrei.net

2 / 67

Outline
I Class Organization
II Introduction to Derivatives
Definitions
Buying and Short-Selling
Continuous Compounding
Forward Contracts
Call Options
Put Options
Moneyness, Put-Call Parity
Options Are Insurance
Various Strategies

4
10
11
17
22
27
33
40
46
56
58

3 / 67

Outline
I Class Organization
II Introduction to Derivatives
Definitions
Buying and Short-Selling
Continuous Compounding
Forward Contracts
Call Options
Put Options
Moneyness, Put-Call Parity
Options Are Insurance
Various Strategies

4
10
11
17
22
27
33
40
46
56
58

4 / 67

Useful Information
I

Class materials:
I
I

My contact information:
I
I
I
I

Slides: Class website on myAnderson


Textbook: Robert McDonald, Derivatives Markets, Pearson Addison
Wesley, second edition, 2006.
Office: C4.20
Office hours: Thursday (Sep 27 - Dec 6), 4-5 pm or by appointment
Phone: (310) 825-3544
Email: daniel.andrei@anderson.ucla.edu

TA: Nimesh Patel


I
I

Office: C4.01
Office hours: Room C4.14, Tuesday (Oct 2 - Dec 11), 5-6 pm or by
appointment
Email: nimesh.patel.2015@anderson.ucla.edu

5 / 67

Schedule: Thursdays, 8:3011:20 AM, Cornell D-313


Week
Week
Week
Week
Week
Week
Week
Week

1:
2:
3:
4:
5:
6:
7:
8:

Sep. 27
Oct. 4
Oct. 11
Oct. 18
Oct. 25
Nov. 1
Nov. 8
Nov. 15

Week 9: Nov. 29
Week 10: Dec. 6

Introduction to derivatives

Chapters 1, 2, 3, 9,
Appendix B

Binomial option pricing

Chapters 10, 11

Black-Scholes, Greeks

Chapters 12, 13

Forwards, futures, and swaps

Chapters 5, 6, 7, 8

Thanksgiving: Thursday, Nov. 22


Volatility risk
Chapters 11, 12, 13
Risk management, other topics Chapters 3, 4, 15, ...

Midterm: Week 7 (Thursday, Nov. 8), 1.5 hours, open book exam

Final exam: Thursday, Dec. 13, D-313, 1.5 hours, open book exam

6 / 67

Evaluation

There will be five quizzes, available via Equiz.me


I

I
I

To register, please follow this link. Please use an email address that
explicitely shows your name (preferably your UCLA Anderson email
address).
Once links are posted, they will be available online for two weeks.
Once available, you can solve them as many times as you want. I
strongly advise working in groups, but you will have to submit your
answers individually.

Grade formula: 40% final exam, 30% midterm, 20% quizzes, 10%
class participation.

7 / 67

In addition to enrolling through the proper authorities,


please send me an email with the following information:

name

program and year in program

major (if any)

your background in finance and mathematics

telephone number

any other information you consider important

8 / 67

Ground Rules

These rules help ensure that no one interferes with the learning of another:
I

I will start promptly and I expect you to be on time

If, because of some unusual circumstance, you come in late, please


enter as quietly as possible

If it is necessary for you to leave early, please sit next to a door

You may leave the room briefly if it is an emergency

As a general rule, you may not use notebook computer or other


handheld computing or communication devices in class

9 / 67

Outline
I Class Organization
II Introduction to Derivatives
Definitions
Buying and Short-Selling
Continuous Compounding
Forward Contracts
Call Options
Put Options
Moneyness, Put-Call Parity
Options Are Insurance
Various Strategies

4
10
11
17
22
27
33
40
46
56
58

10 / 67

What is a Derivative?

Definition
I

Types
I

An agreement between two parties which has a value determined by


the price of something else.
Options, futures, and swaps.

Uses
I
I
I
I

Risk management
Speculation
Reduce transaction costs
Regulatory arbitrage

11 / 67

Three Different Perspectives


Economic
observers:
I Regulators

End-users:

I Researchers

I Corporations

Intermediaries:
I Market-makers
I Traders

End-users:
I Investment managers

End-users:
I Investors

12 / 67

Financial Engineering
I

The construction of a financial product from other products

New securities can be designed using existing securities


Financial engineering principles

I
I
I
I

Facilitate hedging of existing positions


Enable understanding of complex positions
Allow for creation of customized products
Render regulation less effective

Examples:
I

In 2002, Universal Studios issued a $175 million catastrophe bond to


cover its production studios against an earthquake in Southern
California
In 2003, Walt Disney issued a catastrophe bond to cover its theme
park in Japan

13 / 67

Contracts outstanding (millions)

Exchange Traded Derivatives

200

100

85

1,9

1,9

90

95

1,9

2,0

00

05 ,010
2
2,0

source: Bank for International Settlements, Quarterly Review, June 2012.


http://www.bis.org/statistics/extderiv.htm
14 / 67

Notional amount (billions)

Exchange Traded Derivatives (contd)


80,000
60,000
40,000
20,000
0

85

1,9

1,9

90

95

1,9

2,0

00

05

2,0

10

2,0

source: Bank for International Settlements, Quarterly Review, June 2012.


http://www.bis.org/statistics/extderiv.htm

15 / 67

Notional amount (billions)

Over-the-Counter (OTC) Derivatives

600,000

400,000

200,000

98 00 02 04 06 08 10 12
1,9 2,0 2,0 2,0 2,0 2,0 2,0 2,0
source: Bank for International Settlements, Quarterly Review, June 2012.
http://www.bis.org/statistics/derstats.htm
16 / 67

Buying and Selling a Financial Asset


I

Brokers: commissions

Market-makers: bid-ask spread (reflects the perspective of the


market-maker)
The price at which
you can buy
The price at which
you can sell

ask (offer)
bid

What the marketmaker will sell for


What the marketmaker pays

Example: Buy and sell 100 shares of XYZ


I
I
I
I

XYZ: bid=$49.75, ask=$50, commission=$15


Buy: (100 $50) + $15 = $5, 015
Sell: (100 $49.75) $15 = $4, 960
Transaction cost: $5, 015 $4960 = $55

17 / 67

Problem 1.4: ABC stock has a bid price of $40.95 and an


ask price of $41.05. Assume that the brokerage fee is
quoted as 0.3% of the bid or ask price.

a. What amount will you pay to buy 100 shares?


b. What amount will you receive for selling 100 shares?
c. Suppose you buy 100 shares, then immediately sell 100 shares. What is
your round-trip transaction cost?

18 / 67

Problem 1.4: ABC stock has a bid price of $40.95 and an


ask price of $41.05. Assume that the brokerage fee is
quoted as 0.3% of the bid or ask price.
a. What amount will you pay to buy 100 shares?
($41.05 100) + ($41.05 100) 0.003 = $4, 117.32
b. What amount will you receive for selling 100 shares?
($40.95 100) ($40.95 100) 0.003 = $4, 082.72
c. Suppose you buy 100 shares, then immediately sell 100 shares. What is
your round-trip transaction cost?
$4, 117.32 $4, 082.72 = $34.6

18 / 67

Short-Selling
I

When price of an asset is expected to fall


I
I
I
I

First: borrow and sell and asset (get $$)


Then: buy back and return the asset (pay $)
If price fell in the mean time: Profit $ = $$ - $
The lender must be compensated for dividends received

Example: Cash flows associated with short-selling a share of IBM for


90 days. Note that the short-seller must pay the dividend, D, to the
share-lender.

Action
Cash

Day 0
Borrow shares
Sell shares
+S0

Dividend Ex-Day

Day 90
Return shares
Purchase shares
S90

19 / 67

VWs 348% Two-Day Gain Is Pain for Hedge Funds


From the Wall Street Journal, 2008:
In short squeezes, investors who borrowed
and sold stock expecting its value to fall
exit from the trades by buying those
shares, or covering their positions. That
can send a stock upward if shares are hard
to come by. When shares are scarce, that
can push a company-s market
capitalization well beyond a reasonable
valuation. [...] Indeed, the recent stock
gains left Volkswagens market value at
about $346 billion, just below that of the
worlds largest publicly traded corporation,
Exxon Mobil Corp.
20 / 67

Problem 1.6: Suppose you short-sell 300 shares of XYZ


stock at $30.19 with a commission charge of 0.5%.
Supposing you pay commission charges for purchasing the
security to cover the short-sale, how much profit have you
made if you close the short-sale at a price of $29.87?

21 / 67

Problem 1.6: Suppose you short-sell 300 shares of XYZ


stock at $30.19 with a commission charge of 0.5%.
Supposing you pay commission charges for purchasing the
security to cover the short-sale, how much profit have you
made if you close the short-sale at a price of $29.87?
Initially, we will receive the proceeds form the sale of the asset, less the
proportional commission charge:
300 ($30.19) 300 ($30.19) 0.005 = $9, 011.72
When we close out the position, we will again incur the commission
charge, which is added to the purchasing cost:
300 ($29.87) + 300 ($29.87) 0.005 = $9, 005.81
Finally, we receive total profits of: $9, 011.72 $9, 005.81 = $5.91.
21 / 67

Continuous Compounding
I

Terms often used to to refer to interest rates:


I

Effective annual rate r : if you invest $1 today, T years later you will
have
(1 + r )T

Annual rate r , compounded n times per year: if you invest $1


today, T years later you will have


1+

r nT
n

Annualized continuously compounded rate r : if you invest $1


today, T years later you will have

r nT
1+
n
n

e rT lim

22 / 67

Continuous Compounding: Example


I

Suppose you have a zero-coupon bond that matures in 5 years. The


price today is $62.092 for a bond that pays $100.
I

The effective annual rate of return is


1/5

$100
1 = 0.10
$62.092

The continuously compounded rate of return is


ln ($100/$62.092) 0.47655
=
= 0.09531
5
5

The continuously compounded rate of return of 9.53% corresponds to


the effective annual rate of return of 10%. To verify this, observe that
e 0.09531 = 1.10
or


ln (1.10) = ln e 0.09531 = 0.09531


23 / 67

Continous Compounding

When we multiply exponentials, exponents add. So we have


e x e y = e x +y
This makes calculations of average rate of return easier.

When using continuous compounding, increases and decreases are


symmetric.

Moreover, continuously compounded returns can be less than -100%

24 / 67

Problem B.2: Suppose that over 1 year a stock price


increases from $100 to $200. Over the subsequent year it
falls back to $100.

What is the effective return over the first year? What is the continuously
compounded return?

What is the effective return over the second year? The continuously
compounded return?

What do you notice when you compare the first- and second-year returns
computed arithmetically and continuously?

25 / 67

Problem B.2: Suppose that over 1 year a stock price


increases from $100 to $200. Over the subsequent year it
falls back to $100.
I

What is the effective return over the first year? What is the continuously
compounded return?
$200 $100
= 100%
 $100 
$200
= 69.31%
continuously compounded return = ln
$100
effective return =

What is the effective return over the second year? The continuously
compounded return?
$100 $200
= 50%
 $200 
$100
= 69.31%
continuously compounded return = ln
$200
effective return =

What do you notice when you compare the first- and second-year returns
computed arithmetically and continuously?
25 / 67

A World Without Options

Stock Price ($)

Stocks

Payoff ($)

Payoff ($)

Treasury Bills

Stock Price ($)

26 / 67

Forward Contracts

Definition: a binding agreement (obligation) to buy/sell an underlying


asset in the future, at a price set today.
A forward contract specifies:
1. The features and quantity of the asset to be delivered
2. The delivery logistics, such as time, date, and place
3. The price the buyer will pay at the time of delivery

27 / 67

Forward Contracts vs Futures Contracts

Forward and futures contracts are essentially the same except for the
daily resettlement feature of futures contracts, called
marking-to-market.

Because futures are exchange-traded, they are standardized and have


specified delivery dates, locations, and procedures.

Plenty of information is available from: www.cmegroup.com

28 / 67

Reading Price Quotes

Figure 1: Index futures price listings

source: Wall Street Journal, September 11, 2012.


29 / 67

Payoff (Value at Expiration) of a Forward Contract

Every forward contract has both a buyer and a seller.

The term long is used to describe the buyer and short is used to
describe the seller.
Payoff for

I
I

Long forward = Spot price at expiration Forward price


Short forward = Forward price Spot price at expiration

Example: S&R index:


I
I

Today: Spot price = $1,000. 6-month forward price = $1,020


In 6 months at contract expiration: Spot price = $1,050
I
I

Long position payoff = $1,050 - $1,020 = $30


Short position payoff = $1,020 - $1,050 = -$30

30 / 67

Payoff Diagram for Forwards


200

Long forward

Payoff ($)

100

100
$1, 020

200
800

900

1,000

Short forward
1,100

1,200

S&R Index Price ($)

31 / 67

Problem 2.4.a: Suppose you enter in a long 6-month


forward position at a forward price of $50. What is the
payoff in 6 months for prices of $40, $45, $50, $55, and
$60?

32 / 67

Problem 2.4.a: Suppose you enter in a long 6-month


forward position at a forward price of $50. What is the
payoff in 6 months for prices of $40, $45, $50, $55, and
$60?
The payoff to a long forward at expiration is equal to:
Payoff to long forward = Spot price at expiration Forward price
Therefore, we can construct the following table:
Price of asset in 6 months
40
45
50
55
60

Payoff ot the long forward


-10
-5
0
5
10
32 / 67

Call Options

A non-binding agreement (right but not an obligation) to buy an


asset into the future, at a price set today

Preserves the upside potential, while at the same time eliminating the
downside

The seller of a call option is obligated to deliver if asked

33 / 67

Definition and terminology


I

A call option gives the owner the right but not the obligation to buy
the underlying asset at a predetermined price during a predetermined
time period

Strike (or exercise) price: the amount paid by the option buyer for the
asset if he/she decides to exercise

Exercise: the act of paying the strike price to buy the asset

Expiration: the date by which the option must be exercised or


becomes worthless
Exercise style: specifies when the option can be exercised

I
I
I

European-style: can be exercised only at expiration date


American-style: can be exercised at any time before expiration
Bermudan-style: can be exercised during specified periods

34 / 67

Reading Price Quotes

Figure 2: S&P 500 Index Call Options

source: Chicago Board Options Exchange, September 11, 2012.

35 / 67

Call Option Example


I

Consider a call option on the S&R index with 6 months to expiration


and strike price of $1,000.
In six months at contract expiration: if spot price is
I

$1,100 call buyers payoff = $1,100 $1,000 = $100, call sellers


payoff = -$100
$900 call buyers payoff = $0, call sellers payoff = $0

The payoff of a call option is then


CT = max [ST K , 0]

(1)

where K is the strike price, and ST is the spot price at expiration.


I

The option profit is computed as


Call profit = max [ST K , 0] future value of premium

(2)

36 / 67

Diagrams for Purchased Call

200

200

Long forward

Purchased call
100
Profit ($)

Payoff ($)

100

Purchased
call

$95.68

100

100

200

200

$1, 020

800

900

1,000

1,100

S&R Index Price ($)

1,200

800

900

1,000

1,100

1,200

S&R Index Price ($)

37 / 67

Diagrams for Written Call

200

200

100

100
Profit ($)

Payoff ($)

$95.68

100

Written
call

100
Written call

200
800

900

1,000

1,100

S&R Index Price ($)

$1, 020

200
1,200

800

900

1,000

Short forward
1,100

1,200

S&R Index Price ($)

38 / 67

Problem 2.10.a: Consider a call option on the S&R index


with 6 months to expiration and strike price of $1,000.
The future value of the option premium is $95.68. For the
figure below, which plots the profit on a purchased call,
find the S&R index price at which the call option diagram
intersects the x -axis.
200

Profit ($)

100

Purchased
call

$95.68

100

200
800

900

1,000

1,100

1,200

S&R Index Price ($)


39 / 67

Problem 2.10.a: Consider a call option on the S&R index


with 6 months to expiration and strike price of $1,000.
The future value of the option premium is $95.68. For the
figure below, which plots the profit on a purchased call,
find the S&R index price at which the call option diagram
intersects the x -axis.
200

The profit of the long call option is:


max [0, ST $1, 000] $95.68

Profit ($)

100

Purchased
call

$95.68

100

200
800

900

1,000

1,100

1,200

To find the S&R index price at which


the call option diagram intersects the
x -axis, we have to set the above
equation equal to zero. We get
ST = $1, 095.68

S&R Index Price ($)


39 / 67

Put Options

A put option gives the owner the right but not the obligation to sell
the underlying asset at a predetermined price during a predetermined
time period.

The payoff of the put option is


PT = max [K ST , 0]

(3)

The option profit is computed as


Put profit = max [K ST , 0] future value of premium

(4)

40 / 67

Reading Price Quotes

Figure 3: S&P 500 Index Put Options

source: Chicago Board Options Exchange, September 11, 2012.


41 / 67

Diagrams for Purchased Put

200

200

Purchased put

Short forward
100
Profit ($)

Payoff ($)

100

100

100

200

200

Purchased
put

$95.68

$1, 020

800

900

1,000

1,100

S&R Index Price ($)

1,200

800

900

1,000

1,100

1,200

S&R Index Price ($)

42 / 67

200

200

100

100
Profit ($)

Payoff ($)

Diagrams for Written Put

100

Long forward

$95.68

Written
put

100
Written put

200
800

900

$1, 020

200
1,000

1,100

S&R Index Price ($)

1,200

800

900

1,000

1,100

1,200

S&R Index Price ($)

43 / 67

Problem 2.14.a: Suppose the stock price is $40 and the


effective annual interest rate is 8%. Draw payoff and profit
diagrams for a 40-strike put with a premium of $3.26 and
maturity of 1 year.

44 / 67

Problem 2.14.a: Suppose the stock price is $40 and the


effective annual interest rate is 8%. Draw payoff and profit
diagrams for a 40-strike put with a premium of $3.26 and
maturity of 1 year.

FV (premium) = $3.26 (1 + 0.08)


= $3.5208

We get the following payoff and profit


diagram:

40
Payoff (blue) or profit (red) ($)

In order to be able to draw the profit


diagram, we need to find the future
value of the put premium:

30
Payoff

20
Profit

10
0
10

$3.5208

20

40
60
Stock Price ($)

80

44 / 67

Potential Gain and Loss for Forward and Option Positions

Position
Long forward
Short forward
Long call
Short call
Long put
Short put

Maximum Loss
Forward price
Unlimited
FV(premium)
Unlimited
FV(premium)
FV(premium) Strike price

Maximum Gain
Unlimited
Forward Price
Unlimited
FV(premium)
Strike price FV(premium)
FV(premium)

45 / 67

Moneyness

In-the-money option: positive payoff if exercised immediately

At-the-money option: zero payoff if exercised immediately

Out-of-the-money option: negative payoff if exercised immediately

46 / 67

Put-Call Parity
I

Suppose you are buying a call option and selling a put option on a
non-dividend paying stock. Both options have maturity T and strike
price K :
Combined position

Payoff ($)

Payoff ($)

Long call

Short put

Stock Price ($)

Stock Price ($)

47 / 67

Put-Call Parity (contd)


I

Your payoff at maturity is


CT PT = max [ST K , 0] max [K ST , 0]
= max [ST K , 0] + min [ST K , 0]
= ST K

We have two strategies with the same payoff at maturity:


I
I

Buy a call and sell a put, thus paying a premium of Ct Pt today


Buy a share of the stock and borrow PV (K ), thus paying a premium
of St PV (K ) today

Positions that have the same payoff should have the same cost (Law
of one price):
Ct Pt = St PV (K )

(5)

Equation (5) is known as put-call parity, and one of the most


important relations in options.
48 / 67

Put-Call Parity (contd)


I

Parity provides a cookbook for the synthetic creation of options. It


tells us that
Ct = Pt + St PV (K )

(6)

Pt = Ct St + PV (K )

(7)

and that

The first relation says that a call is equivalent to a leveraged position


on the underlying asset, which is insured by the purchase of a put.
The second relation says that a put is equivalent to a short position
on the stock, insured by the purchase of a call

Parity generally fails for American-style options, which may be


exercised prior to maturity.

49 / 67

Why Does the Price of an At-the-Money call Exceed the


Price of an At-the-Money put?
I

Parity shows that the reason for the call being more expensive is the
time value of money:
Ct Pt = K PV (K ) > 0

(8)

A common erroneous explanation is that the profit on a call is


unlimited, while the profit on a put can be no greater than the strike
price, which seems to suggest that the call should be more expensive
than the put.

This argument also seems to suggest that every stock is worth more
than its price!

50 / 67

Problem 3.8: The S&R index price is $1,000 and the


effective 6-month interest rate is 2%. Suppose the
premium on a 6-month S&R call is $109.20 and the
premium on a 6-month put with the same strike price is
$60.18. What is the strike price?

51 / 67

Problem 3.8: The S&R index price is $1,000 and the


effective 6-month interest rate is 2%. Suppose the
premium on a 6-month S&R call is $109.20 and the
premium on a 6-month put with the same strike price is
$60.18. What is the strike price?
This question is a direct application of the Put-Call Parity:
Ct Pt = St PV (K )
K
$109.20 $60.18 = $1, 000
1.02
K = $970.00

51 / 67

Put-Call Parity for Dividend Paying Stocks


I

If the stock is paying dividends over the lifetime of the option, the
put-call parity becomes
Ct Pt = [St PV (Div)] PV (K )

(9)

where PV (Div) is the present value of the stream of dividends paid


on the stock until maturity.
I

Hence, we can write


Ct = Pt + [St PV (Div)] PV (K )

(10)

Pt = Ct [St PV (Div)] + PV (K )

(11)

Equations (10)(11) help us to find maximum and minimum option


prices.

52 / 67

Maximum and Minimum Option Prices: Call Price


Ct
St Ct

Ct max [0, St PV (Div) PV (K )]


St
53 / 67

Maximum and Minimum Option Prices: Call Price


Ct
St Ct

Ct max [0, St PV (Div) PV (K )]


St
53 / 67

Maximum and Minimum Option Prices: Call Price


Ct
St Ct

Ct max [0, St PV (Div) PV (K )]


St
53 / 67

Maximum and Minimum Option Prices: Call Price


Ct
St Ct

Call option
price somewhere here

Ct max [0, St PV (Div) PV (K )]


St
53 / 67

Maximum and Minimum Option Prices: Put Price


Pt

K Pt

Put option
price somewhere here

Pt max [0, PV (K ) St + PV (Div)]

St
54 / 67

Example: Equity-Linked CDs


I

A 1,999 First Union National Bank CD promises to repay in 5.5 years


initial invested amount and 70% of the gain in S&P 500 index (this is
a principal protected equity-linked CD)

Assume $10,000 invested when


S&P 500 = 1,300

Payoff ($)
13,000

Final payoff is

12,000

Payoff of Equity-Linked CD

11,000

Sfinal
1
$10, 000 1 + 0.7 max 0,
1300

i
10,000
9,000

where Sfinal = value of the


S&P 500 after 5.5 years.

$1,300

8,000
600

800

1,000 1,200 1,400 1,600 1,800


S&P 500 Price ($)

55 / 67

Options are Insurance: Insuring a Long Position (Floors)


I

A put option is combined with a position in the underlying asset

Goal: to insure against a fall in the price of the underlying asset


(when one has a long position in that asset)
2,000

2,000
Long S&R Index
Long put

1,000
Payoff ($)

1,000
Payoff ($)

Combined payoff

0
1,000

0
1,000

2,000

2,000
0

500

1,000

1,500

S&R Index Price ($)

2,000

500

1,000

1,500

2,000

S&R Index Price ($)

56 / 67

Options are Insurance: Insuring a Short Position (Caps)


I

A call option is combined with a position in the underlying asset

Goal: to insure against an increase in the price of the underlying asset


(when one has a short position in that asset)
2,000

2,000

Long call

1,000
Payoff ($)

Payoff ($)

1,000

0
1,000

0
Combined payoff

1,000
Short S&R Index

2,000

2,000
0

500

1,000

1,500

S&R Index Price ($)

2,000

500

1,000

1,500

2,000

S&R Index Price ($)

57 / 67

Various Strategies: Payoffs


Bull Spread

Straddle

Profit ($)

Profit ($)

Profit ($)

Collar

Stock Price ($)

Strangle

Butterfly Spread

Ratio Spread

Stock Price ($)

Profit ($)

Profit ($)

Stock Price ($)

Profit ($)

Stock Price ($)

Stock Price ($)

Stock Price ($)

58 / 67

Various Strategies: Positions


Collar

Bull Spread
Klow
Call
Put

KATM

Khigh

Buy

Sell

Strangle
Klow
Call
Put

KATM

Call
Put

KATM

Straddle
Khigh
Sell

Buy

Klow
Call
Put

Butterfly Spread
Khigh
Buy

Buy

Klow

Call
Put

Klow

KATM

Khigh
Buy

Buy

Sell
Sell

KATM

Khigh

Buy
Buy

Ratio Spread
Klow
Call
Put

KATM
Buy

Khigh
Sell (n)

Note that you can achieve the same results with different combinations
(but always at the same cost!)

59 / 67

Various Strategies: Rationales


Bull Spread
I
I
I
I

You believe a stock will


appreciate buy a call option
(forward position insured)
You can lower the cost if you
are willing to reduce your profit
should the stock appreciate
sell a call with higher strike
Surprisingly, you can achieve
the same result by buying a
low-strike put and selling a
high-strike put
Opposite: bear spread

Strangle

Collar
I

A collar is fundamentally a
short position (resembling a
short forward contract)

Often used for insurance when


we own a stock (collared stock)

The collared stock looks like a


bull spread; however, it arises
from a different set of
transactions

Opposite: written collar

Butterfly Spread
I

To reduce the premium of a


straddle, you can buy
out-of-the-money options rather
than at-the-money options.

I
I

Opposite: written strangle

A butterfly spread is a written


straddle to which we add two
options to safeguard the
position: An out-of-the money
put and an out-of-the money
call.
A butterfly spread can be
thought of as a written straddle
for the timid (or for the
prudent!)
Opposite: long iron butterfly

Straddle
I

A straddle can profit from stock


price moves in both directions

The disadvantage is that it has


a high premium because it
requires purchasing two options

Opposite: written straddle

Ratio Spread
I

Ratio spreads involve buying


one option and selling a greater
quantity (n) of an option with a
more out-of-the money strike

The ratio (i.e., 1 by n) is the


number of short options divided
by the number of long options

The options are either both


calls or both puts

It is possible to construct ratio


spreads with zero premium
we can construct insurance that
costs nothing if it is not needed!
60 / 67

A World Without Options

Stock Price ($)

Stocks

Payoff ($)

Payoff ($)

Treasury Bills

Stock Price ($)

61 / 67

Madoff Ran Vast Options Game


From the Wall Street Journal, 2008:
Mr. Madoff told clients he was using a fairly common options-trading strategy to
generate modest but steady returns for more than two decades. The strategy involved
buying stocks, while also trading options in a way designed to limit losses on the shares
[...] People who analyzed client statements said Mr. Madoffs firm couldnt have bought
and sold the options he claimed because those totals would have outstripped total
trading volume those days.

Reproduced from: Gregoriou and Lhabitant, Madoff: A riot of Red Flags, 2009
62 / 67

Collars in Acquisitions: WorldCom/MCI


I

On October 1, 1997, WorldCom Inc. CEO (Bernard Ebbers) sent the


following note to the CEO of MCI (Bert Roberts), and it was also
released through the typical newswires:

I am writing to inform you that this morning WorldCom is publicly


announcing that it will be commencing an offer to acquire all the
outstanding shares of MCI for $41.50 of WorldCom common stock
per MCI share. The actual number of shares of WorldCom common
stock to be exchanged for each MCI share in the exchange offer will
be determined by dividing $41.50 by the 20-day average of the high
and low sales prices for WorldCom common stock prior to the closing
of the exchange offer, but will not be less than 1.0375 shares (if
WorldComs average stock price exceeds $40) or more than 1.2206
shares (if WorldComs average stock price is less than $34).

63 / 67

Collars in Acquisitions: WorldCom/MCI (contd)


The payoff is contingent upon price of WorldComs 20-day average
stock price prior to the closing exchange offer:
50
Value of Offer

Slope = 1.0375

40
Slope = 1.2206

30
$34

20

$40

20
25
30
35
40
45
50
55
WCOMs Average Stock Price at Closing
64 / 67

Example: Another Equity-Linked Note


I

In July 2004, Marshall & Ilsley Corp. (ticker symbol MI) raised $400
million by issuing mandatorily convertible bonds effectively
maturing in August 2007

The bond pays an annual 6.5% coupon and at maturity makes


payments in shares, with the number of shares dependent upon the
firms stock price. The specific terms of the maturity payment are in
the table below
Marshall & Ilsley Share
Price
SMI < 37.32
37.32 SMI 46.28
46.28 < SMI

Number of Shares Paid


to Bondholders
0.6699
$25/SMI
0.5402

65 / 67

Example: Another Equity-Linked Note (contd)


The graph of the maturity payoff should remind us of a written collar:
50
40
Bondholder payoff
Payoff ($)

30
Slope = 0.5402

20

Slope = 0.6699

10
$37.32

$46.28

20
40
60
80
Marshall & Ilsley Stock Price ($)
66 / 67

Positions Consistent With Different Views on the Stock


Price and Volatility Direction

Price Will Fall


No Price View
Price Will Increase

Volatility Will Increase


Buy puts
Buy straddle
Buy calls

No Volatility View
Sell underlying
Do nothing
Buy underlying

Volatility Will Fall


Sell calls
Sell straddle
Sell puts

67 / 67

Option Markets 232D


2. Binomial Option Pricing

Daniel Andrei

Fall 2012

1 / 60

Outline
I Discrete-Time Option Pricing: The Binomial Model

II Call Options
A One-Period Binomial Tree
The Binomial Solution
A Two-Period Binomial Tree
Many Binomial Periods
Self-Financing Strategy

6
7
15
22
27
28

III Put Options

30

IV Uncertainty in the Binomial Model

35

V Risk-Neutral Pricing

41

VI American Options

53

2 / 60

Outline
I Discrete-Time Option Pricing: The Binomial Model

II Call Options
A One-Period Binomial Tree
The Binomial Solution
A Two-Period Binomial Tree
Many Binomial Periods
Self-Financing Strategy

6
7
15
22
27
28

III Put Options

30

IV Uncertainty in the Binomial Model

35

V Risk-Neutral Pricing

41

VI American Options

53

3 / 60

Discrete-Time Option Pricing: The Binomial Model

Until now, we have looked only at some basic principles of option


pricing
I

We examined payoff and profit diagrams, and upper/lower bounds on


option prices
We saw that with put-call parity we could price a put or a call based on
the prices of the combinations on instruments that make up the
synthetic version of the put or call.

What we need to be able to do is price a put or a call without the


other instrument.

In this section, we introduce a simple means of pricing an option.

4 / 60

Discrete-Time Option Pricing: The Binomial Model


(contd)

The approach we take here is called the binomial tree.

The word binomial refers to the fact that there are only two
outcomes (we let the underlying price move to only one of two
possible new prices).

It may appear that this framework oversimplifies things, but the


model can eventually be extended to encompass all possible prices.

5 / 60

Outline
I Discrete-Time Option Pricing: The Binomial Model

II Call Options
A One-Period Binomial Tree
The Binomial Solution
A Two-Period Binomial Tree
Many Binomial Periods
Self-Financing Strategy

6
7
15
22
27
28

III Put Options

30

IV Uncertainty in the Binomial Model

35

V Risk-Neutral Pricing

41

VI American Options

53

6 / 60

The Binomial Option Pricing Model

The binomial option pricing model assumes that, over a period of


time, the price of the underlying asset can move only up or down by a
specified amountthat is, the asset price follows a binomial
distribution:

up

uS

dow

d S

7 / 60

A Simple Example
I

XYZ does not pay dividends and its current price is $41. In one year
the price can be either $59.954 or $32.903, i.e., u = 1.4623 and
d = 0.8025.

Consider a European call option on the stock of XYZ, with a $40


strike an 1 year to expiration. The continuously compounded risk-free
interest rate is 8%.

We wish to determine the option price:


uS = 59.954
S = 41

Cu = max [0, 59.954 40] = 19.954

C =?
dS = 32.903
Cd = max [0, 32.903 40] = 0

8 / 60

A Simple Example (contd)


I

Let us try to find a portfolio that mimics the option (replicating


portfolio).

We have two instruments: shares of stock and a position in bonds


(i.e., borrowing or lending).

To be specific, we wish to find a portfolio consisting of shares of


stock and a dollar amount B in borrowing or lending, such that the
portfolio imitates the option whether the stock rises or falls.

The value of this replicating portfolio at maturity is:


(

59.954 + e 0.08 B
32.903 + e 0.08 B

= 19.954
=0

(1)

9 / 60

A Simple Example (contd)

The unique solution of this system of 2 equations with 2 unknowns is


= 0.738, B = 22.405,

(2)

i.e., buy 0.738 shares of XYZ and borrow $22.405 at the risk-free rate.
I

In computing the payoff for the replicating portfolio, we assume that


we sell the shares at the market price and that we repay the borrowed
amount, plus interest.

Thus, we obtain that the option and the replicating portfolio have the
same payoff: $19.954 if the stock price goes up and $0 if the stock
price goes down.

10 / 60

A Simple Example (contd)

By the law of one price, positions that have the same payoff should
have the same cost.

The price of the option must be


C = 0.738 $41 $22.405 = $7.839

(3)

11 / 60

Arbitraging a Mispriced Option


I

Suppose that the market price for the option is $8 instead of $7.839
(the option is overpriced).

We can sell the option and buy a synthetic option at the same time
(buy low and sell high). The initial cash flow is
$8.00 $7.839 = $0.161

(4)

and there is no risk at expiration:

Written call
0.738 purchased shares
Repay loan of $22.405
Total payoff

Stock Price in 1 Year


$32.903
$59.954
$0
-$19.954
$24.271
$44.225
-$24.271
-$24.271
$0
$0

12 / 60

Arbitraging a Mispriced Option (contd)


I

Suppose that the market price for the option is $7.5 instead of $7.839
(the option is underpriced).

We can buy the option and sell a synthetic option at the same time
(buy low and sell high). The initial cash flow is
$7.839 $7.5 = $0.339

(5)

and there is no risk at expiration:

Purchased call
0.738 short-sold shares
Sell T-bill
Total payoff

Stock Price in 1 Year


$32.903
$59.954
$0
$19.954
-$24.271
-$44.225
$24.271
$24.271
$0
$0

13 / 60

A Remarkable Result

So far we have not specified the probabilities of the stock going up


and down.

In fact, probabilities were not used anywhere in the option price


calculations.

This is a remarkable result: Since the strategy of holding shares


and B bonds replicates the option whichever way the stock
moves, the probability of an up or down movement in the stock
is irrelevant for pricing the option.

14 / 60

The Binomial Solution


I

Suppose that the stock has a continuous dividend yield of , which is


reinvested in the stock. Thus, if you buy one share at time 0 and the
length of a period is h, at time h you will have e h shares.

The up and down movements of the stock price reflect the


ex-dividend price.

We can write the stock price as uS0 when the stock goes up and dS0
when the stock goes down. We can represent the tree for the stock
and the option as follows:
uS0
S0

C1u

C0
dS0
C1d
15 / 60

The Binomial Solution (contd)


I

If the length of a period is h, the interest factor per period is e rh .

A successful replicating portfolio will satisfy


(

S0 u e h + B e rh
S0 d e h + B e rh

= C1u
= C1d

(6)

This is a system of two equations in two unknowns and B. Solving


for and B gives
C u C d

1
= e h S01(ud)

= e rh

C1d uC1u d
ud

= e rh C1u S0 ue h

(7)

16 / 60

The Binomial Solution (contd)


I

Given the expressions (7) for and B, we can derive a simple


formula for the value of the option. The cost of creating the option is
the net cash required to buy the shares and bonds. Thus, the cost of
the option is
C0 = S0 + B
=e

rh

e
C1u

(r )h d

ud

+ C1d

u e (r )h
ud

(8)

Note that if we are interested only in the option price, it is not


necessary to solve for and B; that is just an intermediate step. If
we want to know only the option price, we can use equation (8)
directly.

17 / 60

The Binomial Solution (contd)

The assumed stock price movements, u and d, should not give rise to
arbitrage opportunities. In particular, we require that
d < e (r )h < u

(9)

Note that because is the number of shares in the replicating


portfolio, it can also be interpreted as the sensitivity of the option to
a change in the stock price. If the stock prices changes by $1, then
the option price, S + B, changes by .

18 / 60

The Binomial Solution (contd)


I

In our example, = 0 (XYZ does not pay dividends) and h = 1 (the


length of a period is 1 year).

The solution for and B reduces to

C1u C1d
S0 (ud)
C d uC u d
e r 1 ud 1

(10)

= e r (C1u S0 u)

The option price further simplifies to


C0 = S0 + B = e

u er
d
+ C1d
ud
ud

e
C1u

(11)

19 / 60

Problem 10.1.a: Let S = $100, K = $105, r = 8%


(continuously compounded), T = 0.5, and = 0. Let
u = 1.3, d = 0.8, and the number of binomial periods
n = 1. What are the premium, , and B for a European
call?

20 / 60

Problem 10.1.a: Let S = $100, K = $105, r = 8%


(continuously compounded), T = 0.5, and = 0. Let
u = 1.3, d = 0.8, and the number of binomial periods
n = 1. What are the premium, , and B for a European
call?

Using the formulas given in (7), we calculate the following values:


= 0.5
B = 38.4316
Call price = 11.5684

20 / 60

Problem 10.21: Suppose that u < e (r )h . Show that


there is an arbitrage opportunity. Now suppose that
d > e (r )h . Show again that there is an arbitrage
opportunity.

21 / 60

Problem 10.21: Suppose that u < e (r )h . Show that


there is an arbitrage opportunity. Now suppose that
d > e (r )h . Show again that there is an arbitrage
opportunity.
I

If u < e (r )h , we short a tailed position of the stock and invest the


proceeds at the interest rate. There is an arbitrage opportunity:
t = 0 state = d
state = u
h
Short stock +e S
d S
u S
Lend money e h S +e (r )h S +e (r )h S
Total
0
>0
>0

If d > e (r )h , we buy a tailed position of the stock and borrow at


the interest rate. There is an arbitrage opportunity:
t = 0 state = d
state = u
Buy stock
e h S
+d S
+u S
h
(r
)h
Borrow money
e S e
S e (r )h S
Total
0
>0
>0
21 / 60

A Two-Period Binomial Tree


I

We can extend the previous example to price a 2-year option,


assuming all inputs are the same as before.
S2uu = 87.669
S1u = 59.954
S0 = 41
C0 =?

C1u =?

C2uu = 47.669
S2ud = 48.114

S1d = 32.903

C2ud = 8.114

C1d =?

S2dd = 26.405
C2dd = 0

Note that an up move followed by a down move (S2ud ) generates the


same stock price as a down move followed by an up move (S2du ). This
is called a recombining tree.
22 / 60

A Two-Period Binomial Tree (contd)


I

To price the option when we have two binomial periods, we need to


work backward through the tree.

Suppose that in period 1 the stock price is S1u = $59.954. We can use
equation (11) to derive the option price:
C1u

=e

u er
d
+ C2ud
ud
ud

e
C2uu

= $23.029

(12)

Using equations (10), we can also solve for the composition of the
replicating portfolio:
= 1, B = 36.925

(13)

i.e., buy 1 share of XYZ and borrow $36.925 at the risk-free rate,
which costs 1 $59.954 $36.925 = $23.029.

23 / 60

A Two-Period Binomial Tree (contd)


I

Suppose that in period 1 the stock price is S1d = $32.903. We can use
equation (11) to derive the option price:


C1d = e r C2ud
I

u er
er d
+ C2dd
ud
ud

= $3.187

(14)

Using equations (10), we can also solve for the composition of the
replicating portfolio:
= 0.374, B = 9.111

(15)

i.e., buy 0.374 shares of XYZ and borrow $9.111 at the risk-free rate,
which costs 0.374 $32.903 $9.111 = $3.187.

24 / 60

A Two-Period Binomial Tree (contd)


I

Move backward now at period 0. The stock price is S0 = 41. We can


use equation (11) to derive the option price:
C0 = e

u er
d
+ C1d
ud
ud

e
C1u

= $10.737

(16)

Using equations (10), we can also solve for the composition of the
replicating portfolio:
= 0.734, B = 19.337

(17)

i.e., buy 0.734 shares of XYZ and borrow $19.337 at the risk-free
rate, which costs 0.734 $41 $19.337 = $10.737.

25 / 60

A Two-Period Binomial Tree (contd)


I

The two-period binomial tree with the option price at each node as
well as the details of the replicating portfolio is:
S2uu = $87.669
S1u = $59.954

C2uu = $47.669

C1u = $23.029
S0 = $41
C0 = $10.737
= 0.734
B = $19.337

=1
B = $36.925
S1d = $32.903

S2ud = $48.114
C2ud = $8.114

C1d = $3.187
= 0.374
B = $9.111

S2dd = $26.405
C2dd = $0
26 / 60

Many Binomial Periods: Three-Period Example


I
I

Once we understand the two-period option it is straightforward to


value an option using more than two binomial periods.
The important principle is to work backward through the tree:
uuu

= $128.198

uuu

= $88.198

S3
uu
S2
uu
C2

=1

B = $36.925

C1 = $26.258
S0 = $41
= 0.798
B = $19.899

= $50.745

S1 = $59.954

C0 = $12.799

= $87.669

= 0.981
B = $32.580
d

S1 = $32.903
d

C1 = $4.685

C3

ud
S2

= $48.114

ud
C2

= $11.925

uud

= $70.356

uud

= $30.356

S3
C3

= 0.956
B = $34.086

= 0.549

dd
S2

= $26.405

B = $13.390

dd
C2

= $0

udd

= $38.612

udd

= $0

ddd

= $21.191

ddd

= $0

S3
C3

=0
B = $0

S3
C3

27 / 60

Self-Financing Strategy

In the two-period binomial tree example, suppose that the stock


moves from S0 = $41 to S1u = $59.954.
The replicating portfolio should be modified as follows:
1. Buy 1 0.734 = 0.266 shares of XYZ (increase the XYZ position from
0.734 shares to 1 share), which costs 0.266 $59.954 = $15.977.
2. Increase borrowing from $19.337 e 0.08 = $20.947 to $36.925, which
yields $15.977.

The amount necessary to buy shares is equal to the amount obtained


from increased borrowing.

Modifying the portfolio does not require additional cash. Thus, the
replicating portfolio is self-financing.

28 / 60

Self-Financing Strategy (contd)


I

If the stock moves from S0 = $41 to S1d = $32.903, the replicating


portfolio should be modified as follows:
1. Sell 0.734 0.374 = 0.360 shares of XYZ (decrease the XYZ position
from 0.734 shares to 0.374 shares), which yields
0.360 $32.903 = $11.836.
2. Decrease borrowing from $19.337 e 0.08 = $20.947 to $9.111, which
costs $11.836.

The amount obtained from selling shares is equal to the amount


necessary to decrease borrowing.

Modifying the portfolio does not require additional cash. Once again,
the replicating portfolio is self-financing.

29 / 60

Outline
I Discrete-Time Option Pricing: The Binomial Model

II Call Options
A One-Period Binomial Tree
The Binomial Solution
A Two-Period Binomial Tree
Many Binomial Periods
Self-Financing Strategy

6
7
15
22
27
28

III Put Options

30

IV Uncertainty in the Binomial Model

35

V Risk-Neutral Pricing

41

VI American Options

53

30 / 60

Put Options

We compute put option prices using the same stock price tree and in
the same way as call option prices.

The only difference with an European put option occurs at expiration:


Instead of computing the price as max [0, S K ], we use
max [0, K S].

Here is a two-period binomial tree for an European put option with a


$40 strike:

31 / 60

Put Options (contd)


S2uu = $87.669
S1u = $59.954

P2uu = $0

P1u = $0
S0 = $41

=0

P0 = $3.823

B = $0

= 0.266

S1d = $32.903

B = $14.749

P1d = $7.209

S2ud = $48.114
P2ud = $0

= 0.626
B = $27.814

S2dd = $26.405
P2dd = $13.595

The proof that the replicating portfolio is self-financing is left as an


exercise.
32 / 60

Put Options (Using the Parity Relationship)


I

For non-dividend paying stocks, the basic parity relationship for


European options with the same strike price and time to expiration is
Ct Pt = St PV (strike price)

(18)

We can use this relationship to find the put price at all nodes:
S2uu = 87.669
C2uu = 47.669
S1u = 59.954
S0 = 41

C1u

= 23.029

P1u

=0

S1d
C1d

= 32.903

S2ud = 48.114
C2ud = 8.114

C0 = 10.737
P0 = 3.823

P2uu = 0

= 3.187

P1d = 7.209

P2ud = 0
S2dd = 26.405
C2dd = 0
P2dd = 13.595
33 / 60

Problem 10.1.b: Let S = $100, K = $105, r = 8%


(continuously compounded), T = 0.5, and = 0. Let
u = 1.3, d = 0.8, and the number of binomial periods
n = 1. What are the premium, , and B for a European
put?

34 / 60

Problem 10.1.b: Let S = $100, K = $105, r = 8%


(continuously compounded), T = 0.5, and = 0. Let
u = 1.3, d = 0.8, and the number of binomial periods
n = 1. What are the premium, , and B for a European
put?

Using the formulas given in (7), we calculate the following values:


= 0.5
B = 62.4513
Put price = 12.4513

34 / 60

Outline
I Discrete-Time Option Pricing: The Binomial Model

II Call Options
A One-Period Binomial Tree
The Binomial Solution
A Two-Period Binomial Tree
Many Binomial Periods
Self-Financing Strategy

6
7
15
22
27
28

III Put Options

30

IV Uncertainty in the Binomial Model

35

V Risk-Neutral Pricing

41

VI American Options

53

35 / 60

Uncertainty in the Binomial Model

A natural measure of uncertainty about the stock return is the


annualized standard deviation of the continuously compounded
stock return, which we will denote by .

If we split the year into n periods of length h (so that h = 1/n), the
standard deviation over the period of length h, h , is (assuming
returns are uncorrelated over time)

(19)
h = h

In other words, the standard deviation of the stock return is


proportional to the square root of time.

36 / 60

Uncertainty in the Binomial Model (contd)


I

We incorporate uncertainty into the binomial tree by modeling the


up and down moves of the stock price. Without uncertainty, the
stock price next period must equal:
St+h = St e (r )h

(20)

To interpret this, without uncertainty, the rate of return on the stock


must be the risk-free rate. Thus, the stock price must rise at the
risk-free rate less the dividend yield, r .

We now model the stock price evolution as


uSt = St e (r )h e +
dSt = St e (r )h e

h
h

(21)

37 / 60

Uncertainty in the Binomial Model (contd)


I

We can rewrite this as


u = e (r )h+
d = e (r )h

h
h

(22)

Return has two parts, one of which is certain [(r ) h], and the other
of which
and generates the up and down stock price
 isuncertain

moves h .

Note that if we set volatility equal to zero, we are back to (20) and we
have St+h = uSt = dSt = St e (r )h . Zero volatility does not mean
that prices are fixed; it means that prices are known in advance.

38 / 60

Uncertainty in the Binomial Model (contd)


I

In our example we assumed that u = 1.4623 and d = 0.8025. These


correspond to an annual stock price volatility of 30%:
u = e (0.080)1+0.3
d = e (0.080)10.3

= 1.4623

= 0.8025

(23)

We will use equations (22) to construct binomial trees. This approach


(called the forward tree approach) is very convenient because it
never violates the no arbitrage restriction
d < e (r )h < u

39 / 60

Problem 10.19: For a stock index, S = $100, = 30%,


r = 5%, = 3%, and T = 3. Let n = 3.

What is the price of a European call option with a strike of $95?

What is the price of a European put option with a strike of $95?

40 / 60

Problem 10.19: For a stock index, S = $100, = 30%,


r = 5%, = 3%, and T = 3. Let n = 3.
I

What is the price of a European call option with a strike of $95?


The price of a European call
option with a strike of 95 is
$24.0058

What is the price of a European put option with a strike of $95?


The price of a European put
option with a strike of 95 is
$14.3799

40 / 60

Outline
I Discrete-Time Option Pricing: The Binomial Model

II Call Options
A One-Period Binomial Tree
The Binomial Solution
A Two-Period Binomial Tree
Many Binomial Periods
Self-Financing Strategy

6
7
15
22
27
28

III Put Options

30

IV Uncertainty in the Binomial Model

35

V Risk-Neutral Pricing

41

VI American Options

53

41 / 60

Risk-Neutral Pricing
I

There is a probabilistic interpretation of the binomial solution for the


price of an option (equation 8, restated below):
C0 = e rh
(r )h

u e (r )h
e (r )h d
+ C1d
C1u
ud
ud
(r )h

(24)

The terms e udd and ueud


follows from inequality 9).

Thus, we can interpret these terms as probabilities. Equation (8) can


be written as
h

sum to 1 and are both positive (this

C0 = e rh p C1u + (1 p ) C1d
I

(25)

This expression has the appearance of an expected value discounted


(r )h
at the risk-free rate. Thus, we will call p e udd the risk-neutral
probability of an increase in the stock price.
42 / 60

Risk-Neutral Pricing (contd)

Pricing options using risk-neutral probabilities can be done in one


step, no matter how large the number of periods. In our example,
p = 0.4256. For the one-period European call option we have:
Call Price in 1 Year
$19.954
$0

Probability
0.4256
0.5744

The call price is


C0 = e 0.08 (0.4256 $19.954 + 0.5744 $0) = $7.839

(26)

43 / 60

Risk-Neutral Pricing (contd)


I

For the two-period European call option we have:


Call Price in 2 Years
Probability
$47.669
p 2 = 0.1811
$8.114
2p (1 p ) = 0.4889
$0
(1 p )2 = 0.3300
The call price is
C0 = e 0.082 (0.1811 $47.669 + 0.4889 $8.114 + 0.3300 $0)
= $10.737

(27)

The probability of reaching any given node is the probability of one


path reaching that node times the number of paths reaching that
node. For example, the probability of reaching the node
S2ud = $48.114 is 2p (1 p ).

It can be easily verified that the sum of probabilities in the table


above is 1.
44 / 60

Risk-Neutral Pricing (contd)


I

For the three-period European call option we have:


Call Price in 3 Years
Probability
$88.198
p 3 = 0.0771
2
$30.356
3p (1 p ) = 0.3121
$0
3p (1 p )2 = 0.4213
$0
(1 p )3 = 0.1896
The call price is
C0 = e 0.083

3
X

h
i
3!
p k (1 p )3k max S0 u k d 3k K , 0
k! (3 k)!
(28)
k =0

= $12.799
I

It can be easily verified that the sum of probabilities in the table


above is 1.

It is left as an exercise to find the price of the two-period European


put using risk-neutral probabilities.
45 / 60

Risk-Neutral Pricing (contd)

For an arbitrary number of periods n, the price of an European call


option is given by
C0 = e rn

n
X

h
i
n!
p k (1 p )nk max S0 u k d nk K , 0 (29)
k! (n k)!
k=0

We will use this formula later on when talking about Black-Scholes:


when the number of steps becomes great enough the price of the
option appear to approach a limiting value. This value is given by the
Black-Scholes formula.

46 / 60

Problem 11.12: Let S = $100, = 0.3, r = 0.08, T = 1,


and = 0. Use equation (29) to compute the risk-neutral
probability of reaching a terminal node and the price at
that node for n = 3. Plot the risk-neutral distribution of
year-1 stock prices.

47 / 60

Problem 11.12: Let S = $100, = 0.3, r = 0.08, T = 1,


and = 0. Use equation (29) to compute the risk-neutral
probability of reaching a terminal node and the price at
that node for n = 3. Plot the risk-neutral distribution of
year-1 stock prices.
For n = 3, u and d are calculated as
follows:

u = e (0.080)1/3+0.3 1/3 = 1.2212

d = e (0.080)1/30.3 1/3 = 0.8637


It follows that p = 0.4568, and
nk
0
1
2
3

Stock price
182.14
128.81
91.10
64.43

Probability
0.0953
0.3400
0.4044
0.1603

0.4
0.3
0.2
0.1
64.43

91.10 128.81 182.14

47 / 60

Understanding Risk-Neutral Pricing

A risk-neutral investor is indifferent between a sure thing and a risky


bet with an expected payoff equal to the value of the sure thing.

A risk-averse investor prefers a sure thing to a risky bet with an


expected payoff equal to the value of the sure thing.

Formula (25) suggests that we are discounting at the risk-free rate,


even though the risk of the option is at least as great as the risk of
the stock.

Thus, the option pricing formula, equation (25), can be said to price
options as if investors are risk-neutral.

Is this option pricing consistent with standard discounted cash


flow calculations?

48 / 60

Understanding Risk-Neutral Pricing (contd)


I

Assume, as in our example, that a stock does not pay dividends and
the length of a period is 1 year ( = 0 and h = 1).

Risk-Neutral Pricing
I

The risk-free rate is the discount


rate for any asset including the
stock:
S0 = e r [p uS0 + (1 p ) dS0 ]
Solving for p gives us
er d
p =
ud

Standard DCF calculation


I

Suppose that the continuously


compounded expected return on
the stock is . If p is the true
probability of the stock going up, p
must be consistent with u, d, and
.
S0 = e [puS0 + (1 p) dS0 ]

Solving for p gives us


p=

e d
ud

49 / 60

Understanding Risk-Neutral Pricing (contd)


Risk-Neutral Pricing
I

We discount the option payoff at


the risk-free rate, r .

Standard DCF calculation


I

At what rate do we discount the


option payoff? Since an option is
equivalent to holding a portfolio
consisting of shares of stock and
B bonds, the expected return on
this portfolio is
e =

The option price is


h
i
C0 = e r p C1u + (1 p ) C1d

B
S0
e +
er
S0 + B
S0 + B

where is the discount rate for the


option.
The option price is
h
i
C0 = e pC1u + (1 p) C1d

50 / 60

Understanding Risk-Neutral Pricing (contd)

Are these two prices the same? Yes (proof left as an exercise).

Note that it does not matter whether we have the correct value of
to start with.

Any consistent pair of and will give the same option price.

Risk-neutral pricing is valuable because setting = r results in the


simplest pricing procedure.

51 / 60

Understanding Risk-Neutral Pricing (contd)

We need to emphasize that at no point are we assuming that


investors are risk-neutral.

Rather, risk-neutral pricing is an interpretation of the formulas


above.

52 / 60

Outline
I Discrete-Time Option Pricing: The Binomial Model

II Call Options
A One-Period Binomial Tree
The Binomial Solution
A Two-Period Binomial Tree
Many Binomial Periods
Self-Financing Strategy

6
7
15
22
27
28

III Put Options

30

IV Uncertainty in the Binomial Model

35

V Risk-Neutral Pricing

41

VI American Options

53

53 / 60

American Options
I

An European option can only be exercised at the expiration date,


whereas an American option can be exercised at any time.

Because of this added flexibility, an American option must always be


at least as valuable as an otherwise identical European option:
CAmer (S, K , T ) CEur (S, K , T )
PAmer (S, K , T ) PEur (S, K , T )

Combining these statements, together with the maximum and


minimum option prices (from the first Section), gives us
S CAmer (S, K , T ) CEur (S, K , T ) max [0, S PV (Div ) PV (K )]
K PAmer (S, K , T ) PEur (S, K , T ) max [0, PV (K ) S + PV (Div )]

54 / 60

American Call

An American-style call option on a nondividend-paying stock should


never be exercised prior to expiration (proof in class).

For an American call on a dividend-paying stock it might be beneficial


to exercise the option prior to expiration (by exercising the call, the
owner will be entitled to dividend payments that she would not have
otherwise received).

Consider the previous example, with one exception: = 0.065, i.e.,


XYZ stock has a continuous dividend yield of 6.5% per year.

55 / 60

American Call (contd)


I

Because of dividends, early exercise


is optimal at the node where the
stock price is $76.982.
S1u = $56.181
C1u = $18.255
u
C1,NO
u
C1,EX

S3uuu = $105.485
S2uu = $76.982
C2uu = $36.982
uu
C2,NO
= $35.213
uu
C2,EX
= $36.982

= $18.255

S2ud = $42.249

= $16.181

C2ud = $7.029

C0,NO = $8.635

S1d = $30.833

ud
C2,NO
= $7.029

C0,EX = $1

C1d = $2.761

ud
C2,EX
= $2.249

S0 = $41
C0 = $8.635

d
C1,NO

= $2.761

d
C1,EX

= $0

C,NO = value of call if not exercised


C,EX = value of call if exercised

C3uuu = $65.485

S2dd = $23.187
C2dd
dd
C2,NO

= $0

dd
C2,EX

= $0

S3uud = $57.892
C3uud = $17.892

S3udd = $31.772
C3udd = $0

= $0
S3ddd = $17.437
C3ddd = $0

56 / 60

American Put

When the underlying stock pays no dividend, a call will not be


early-exercised, but a put might be.

Suppose a company is bankrupt and the stock price falls to zero.


Then a put that would not be exercised until expiration will be worth
PV (K ), which is smaller than K for a positive interest rate.

Therefore, early exercise would be optimal in order to receive the


strike price earlier.

This can also be shown by using a parity argument.

Consider the previous example, with = 0.065.

57 / 60

American Put (contd)


I

Early exercise is optimal at the


node where the stock price is
$23.187.
S1u = $56.181
P1u = $2.314
S0 = $41
P0 = $6.546

u
P1,NO
u
P1,EX

S3uuu = $105.485
S2uu = $76.982
P2uu = $0
uu
P2,NO
= $0
uu
P2,EX
= $0

= $2.314

S2ud = $42.249

= $0

P2ud = $4.363

P0,NO = $6.546

S1d = $30.833

ud
P2,NO
= $4.363

P0,EX = $0

P1d = $10.630

ud
P2,EX
= $0

d
P1,NO

= $10.630

d
P1,EX

= $9.167

P,NO = value of put if not exercised


PC,EX = value of put if exercised

P3uuu = $0

S2dd = $23.187
P2dd
dd
P2,NO

= $16.813

dd
P2,EX

= $16.813

S3uud = $57.892
P3uud = $0

S3udd = $31.772
P3udd = $8.228

= $15.197
S3ddd = $17.437
P3ddd = $22.563

58 / 60

Problem 10.20: For a stock index, S = $100, = 30%,


r = 5%, = 3%, and T = 3. Let n = 3.

What is the price of an American call option with a strike of $95?

What is the price of an American put option with a strike of $95?

59 / 60

Problem 10.20: For a stock index, S = $100, = 30%,


r = 5%, = 3%, and T = 3. Let n = 3.
I

What is the price of an American call option with a strike of $95?


The price of an American call
option with a strike of 95 is
$24.1650

What is the price of an American put option with a strike of $95?


The price of an American put
option with a strike of 95 is
$15.2593

59 / 60

Understanding Early Exercise


I

In deciding whether to early-exercise an option, the option holder


compares the value of exercising immediately with the value of
continuing to hold the option.

Consider the cost and benefits of early exercise for a call option and a
put option. By exercising, the option holder

Call Option

Put Option

Receives the stock and therefore


receives future dividends

Dividends are lost by giving up the


stock

Bears the interest cost of paying


the strike price prior to expiration

Receives the strike price sooner


rather than later

Loses the insurance implicit in the


call (the option holder is protected
against the possibility that the
stock price will be less than the
strike price at expiration).

Loses the insurance implicit in the


put (the option holder is protected
against the possibility that the
stock price will be more than the
strike price at expiration)
60 / 60

Option Markets 232D


3. Black-Scholes Formula and the Greeks
Daniel Andrei

Fall 2012

1 / 74

Outline
I From Binomial Trees to the Black-Scholes Option Pricing Formula
Discrete Time vs Continuous Time
The Limiting Case of the Binomial Formula
Lognormality and the Binomial Model
Black-Scholes Assumptions
Inputs in the Binomial Model and in Black-Scholes
II Black-Scholes Formula
Black-Scholes Formula for a European Call Option
Black-Scholes Formula for a European Put Option
III Implied Volatility
IV WSJ reading
V Market-Maker Risk and Delta-Hedging
VI Option Greeks
VII Gamma-Neutrality
VIII Calendar Spreads
IX Appendix: Formulas for Option Greeks

3
4
5
11
14
15
16
17
23
27
33
38
46
60
66
70
2 / 74

Outline
I From Binomial Trees to the Black-Scholes Option Pricing Formula
Discrete Time vs Continuous Time
The Limiting Case of the Binomial Formula
Lognormality and the Binomial Model
Black-Scholes Assumptions
Inputs in the Binomial Model and in Black-Scholes
II Black-Scholes Formula
Black-Scholes Formula for a European Call Option
Black-Scholes Formula for a European Put Option
III Implied Volatility
IV WSJ reading
V Market-Maker Risk and Delta-Hedging
VI Option Greeks
VII Gamma-Neutrality
VIII Calendar Spreads
IX Appendix: Formulas for Option Greeks

3
4
5
11
14
15
16
17
23
27
33
38
46
60
66
70
3 / 74

Discrete Time vs Continuous Time


I

We refer to the structure of the binomial model as discrete time,


which means that time moves in distinct increments

This is much like looking at a calendar and observing only months,


weeks, or days

We know that time moves forward at a rate faster than one day at a
time: hours, minutes, seconds, fractions of seconds, and fractions and
fractions of seconds

When we talk about time moving in the tiniest increments, we are


talking about continuous time.
Discrete time world
Binomial model

Continuous time world


Black-Scholes model

4 / 74

The Limiting Case of the Binomial Formula

An obvious objection to the binomial calculations thus far is that the


stock can only have a few different values at expiration. It seems
unlikely that the option price calculation will be accurate.

The solution to this problem is to divide the time to expiration into


more periods, generating a more realistic tree.

To illustrate how to do this, we will re-examine the 1-year European


call option, which has a $40 strike and initial stock price $41.

Let there be 3 binomial periods. Since it is a 1-year call, this means


that the length of a period is h = 1/3. We will assume that other
inputs stay the same, so r = 0.08 and = 0.3.

5 / 74

The Limiting Case of the Binomial Formula (contd)


I

Since the length of the binomial period is shorter, u and d are closer
to 1 than before (1.2212 and 0.8637 as opposed to 1.4623 and 0.8025
with h = 1).

The risk-neutral probability of the stock price going up in a period is


p =

e (0.080)1/3 0.8637
= 0.4568
1.2212 0.8637

The binomial model implicitly assigns probabilities to the various


nodes. The risk-neutral probability for each final period node,
together with the call value, is:
Call Price in 1 Year (3 periods)
$34.678
$12.814
$0
$0

Probability
p 3 = 0.0953
3p 2 (1 p ) = 0.34
3p (1 p )2 = 0.4044
(1 p )3 = 0.1603
6 / 74

The Limiting Case of the Binomial Formula (contd)


I

Thus, the price of the European call option is given by


C0 = e 0.081 (0.0953 $34.678 + 0.34 $12.814)
= $7.0739

We can vary the number of binomial steps, holding fixed the time to
expiration,T . The general formula is
C0 = e

rT

n
X

h
i
n!
p k (1 p )nk max S0 u k d nk K , 0 (1)
k! (n k)!
k=0

(we also need to modify u, d, and p at each time).

7 / 74

The Limiting Case of the Binomial Formula (contd)


I

The following table computes binomial call option prices, using the
same inputs as before.
Number of steps (n)
1
2
3
4
10
50
100
500

Binomial Call Price ($)


$7.839
$7.162
$7.074
$7.160
$7.065
$6.969
$6.966
$6.960
$6.961

Changing the number of steps changes the option price, but once the
number of steps becomes great enough we appear to approach a
limiting value for the price, given by the Black-Scholes formula.
8 / 74

The Limiting Case of the Binomial Formula (contd)


This can be seen graphically below:

Call Price

7.5

7
0

6
8 10 12 14
Number of Steps (n)

16

18

20

9 / 74

Problem 12.2: Using the BinomCall function, compute


the binomial approximations for the following call option:
S0 = $41, K = $40, = 0.3, r = 0.08, T = 0.25 (3
months), and = 0. Be sure to compute prices for
n = 8, 9, 10, 11, and 12. What do you observe about the
behavior of the binomial approximation?

10 / 74

Problem 12.2: Using the BinomCall function, compute


the binomial approximations for the following call option:
S0 = $41, K = $40, = 0.3, r = 0.08, T = 0.25 (3
months), and = 0. Be sure to compute prices for
n = 8, 9, 10, 11, and 12. What do you observe about the
behavior of the binomial approximation?
N
8
9
10
11
12

Call Price
3.464
3.361
3.454
3.348
3.446

The observed values are slowly converging towards the Black-Scholes value
(3.399). Please note that the binomial solution oscillates as it approaches
the Black-Scholes value.
10 / 74

Lognormality and the Binomial Model

The lognormal distribution is the probability distribution that arises


from the assumption that continuously compounded returns on the
stock are normally distributed.

In a binomial tree, as we increase the number of periods until


expiration, continuously compounded returns approach a normal
distribution.

We can plot probabilities of outcomes (of stock returns) from the


binomial tree for different values of n (2, 3, 6, and 10), as shown in
the following figure.

The 10-period binomial tree approaches fairly well the normal


distribution.

11 / 74

Lognormality and the Binomial Model (contd)


0.4

0.6

0.3
0.5
0.2
0.4

0.1
-0.22

0.38

0.3

-0.44

-0.09

0.25

0.60

0.2

0.2
0.1

0.1
0

-0
.8
-0 7
.6
-0 8
.4
-0 9
.3
-0 0
.1
0. 1
0
0. 8
2
0. 7
4
0. 6
65
0.
8
1. 4
03

.1
6
0.
08
0.
32
0.
57
0.
81

-0

.4
-0

-0

.6

12 / 74

Problem 11.13: Let S = $100, = 0.30, r = 0.08, t = 1,


and = 0. Use equation (1) to compute the probability of
reaching a terminal node and the price at that node and
plot the risk-neutral distribution of year-1 stock prices for
n = 50. What is the risk-neutral probability that S1 < $80?
S1 > $120?

13 / 74

Problem 11.13: Let S = $100, = 0.30, r = 0.08, t = 1,


and = 0. Use equation (1) to compute the probability of
reaching a terminal node and the price at that node and
plot the risk-neutral distribution of year-1 stock prices for
n = 50. What is the risk-neutral probability that S1 < $80?
S1 > $120?
For n = 50, we have u = 1.0450,
d = 0.9600, and p = 0.4894. We get
the following diagram:
To obtain the required probabilities we
sum all probabilities for which the final
stock price is below 80 or above 120
respectively. We obtain

0.1

0.05

Pr (S < 80) = 0.2006


Pr (S > 120) = 0.2829

0
0

200 400 600 800


13 / 74

Black-Scholes Assumptions

Assumptions about stock return distribution


I

Continously compounded returns on the stocks are normally distributed


and independent over time
The volatility of continuously compounded returns is known and
constant
Future dividends are known, either as dollar amount or as a fixed
dividend yield

Assumptions about the economic environment


I
I
I

The risk-free rate is known and constant


There are no transaction costs or taxes
It is possible to short-sell costlessly and to borrow at the risk-free rate

14 / 74

Inputs in the Binomial Model and in Black-Scholes


I

There are seven inputs to the binomial model and six inputs to the
Black-Scholes model:
Inputs
Current price of the stock, S0
Strike price of the option, K
Volatility of the stock,
Continuously compounded risk-free
interest rate, r
Time to expiration, T
Dividend yield on the stock,
Number of binomial periods, n

Binomial
Model
X
X
X
X

BlackScholes
X
X
X
X

X
X
X

X
X

15 / 74

Outline
I From Binomial Trees to the Black-Scholes Option Pricing Formula
Discrete Time vs Continuous Time
The Limiting Case of the Binomial Formula
Lognormality and the Binomial Model
Black-Scholes Assumptions
Inputs in the Binomial Model and in Black-Scholes
II Black-Scholes Formula
Black-Scholes Formula for a European Call Option
Black-Scholes Formula for a European Put Option
III Implied Volatility
IV WSJ reading
V Market-Maker Risk and Delta-Hedging
VI Option Greeks
VII Gamma-Neutrality
VIII Calendar Spreads
IX Appendix: Formulas for Option Greeks

3
4
5
11
14
15
16
17
23
27
33
38
46
60
66
70
16 / 74

Black-Scholes Formula for a European Call Option


I

The Black-Scholes formula for a European call option on a stock that


pays dividends at the continuous rate is
C0 (S0 , K , , r , T , ) = S0 e T N (d1 ) Ke rT N (d2 )

(2)

where


S0
K

+ r + 21 2 T

d1 =
T

d2 = d1 T
ln

(3)
(4)

N (x ) is the cumulative normal distribution function, which is the


probability that a number randomly drawn from a standard normal
distribution will be less than x .

17 / 74

Black-Scholes Formula for a European Call Option (contd)

Let S0 = $41, K = $40, = 0.3, r = 0.08, T = 1 year, and = 0.


Computing the Black-Scholes call price, we obtain
C0 = $41 e 0 N (0.49898) $40 e 0.08 N (0.19898)
= $41 e 0 0.6911 $40 e 0.08 0.5789
= $6.961

Note that this result corresponds to the limit obtained from the
binomial model.

18 / 74

Black-Scholes Formula for a European Call Option (contd)


The following figure plots Black-Scholes call option prices (today and
at expiration) for stock prices ranging from $20 to $60.
25
20
Call Price

15
10
5
0
20

30

40
50
Stock Price

60

19 / 74

A Remarkable Result

In the binomial model, we have not specified the probabilities of the


stock going up and down (which would give us the expected return of
the stock).

The expected return on the stock does not appear in the


Black-Scholes formula either.

This raises a question: Consider a stock with a higher beta (and


hence having a higher expected return). A call option on this stock
would have a higher probability of settlement in-the-money, hence a
higher option price. Why is this not the case?

20 / 74

A Remarkable Result (contd)


I

The Black-Scholes formula (2) provides the answer. This formula


shows the composition of the replicating portfolio, which in the
binomial case was
C0 = S0 + B

We can easily identify (the position in the risky asset) and B (the
dollar amount of borrowing or lending) in the Black-Scholes formula:
= e T N (d1 )

(5)

B = Ke rT N (d2 )

(6)

21 / 74

A Remarkable Result (contd)

If S is the stock beta, then the option beta is


Option =

S
S0 + B

(7)

A high stock beta implies a high option beta, so the discount rate for
the expected payoff of the option is correspondingly greater.

The net resultone of the key insights from the Black-Scholes


analysisis that beta is irrelevant: The larger average payoff to
options on high beta stocks is exactly offset by the larger
discount rate.

22 / 74

Black-Scholes Formula for a European Put Option

The Black-Scholes formula for a European put option is


P0 (S0 , K , , r , T , ) = S0 e T N (d1 ) + Ke rT N (d2 )

(8)

where d1 and d2 are given by equations (3) and (4).


I

Put-call parity must hold:


P0 (S0 , K , , r , T , ) = C0 (S0 , K , , r , T , ) + Ke rT Se T

(9)

This follows from the formulas (2) and (8), together with the fact
that for any x , N (x ) = 1 N (x ).

23 / 74

Black-Scholes Formula for a European Put Option (contd)


I

Let S0 = $41, K = $40, = 0.3, r = 0.08, T = 1 year, and = 0.


Computing the Black-Scholes put price, we obtain
P0 = $41 e 0 N (0.49898) + $40 e 0.08 N (0.19898)
= $41 e 0 0.3089 + $40 e 0.08 0.4211
= $2.886

In the binomial model, if we fix the number of periods to n = 500, we


obtain a price of $2.885.

Computing the price using put-call parity (equation 9) yields


P0 = $6.961 + $40e 0.08 $41
= $2.886

24 / 74

Black-Scholes Formula for a European Put Option (contd)


The following figure plots Black-Scholes put option prices (today and
at expiration) for stock prices ranging from $20 to $60.
20
15
Put Price

10
5
0
20

30

40
50
Stock Price

60

25 / 74

Problem 12.20: Let S = $100, K = $90, = 30%,


r = 8%, = 5%, and T = 1.

a. What is the Black-Scholes call price?


b. Now price a put where S = $90, K = $100, = 30%, r = 5%, = 8%,
and T = 1.

26 / 74

Problem 12.20: Let S = $100, K = $90, = 30%,


r = 8%, = 5%, and T = 1.
a. What is the Black-Scholes call price?
The Black-Scholes call price is
$17.70

b. Now price a put where S = $90, K = $100, = 30%, r = 5%, = 8%,


and T = 1.
The Black-Scholes put price is
$17.70

26 / 74

Outline
I From Binomial Trees to the Black-Scholes Option Pricing Formula
Discrete Time vs Continuous Time
The Limiting Case of the Binomial Formula
Lognormality and the Binomial Model
Black-Scholes Assumptions
Inputs in the Binomial Model and in Black-Scholes
II Black-Scholes Formula
Black-Scholes Formula for a European Call Option
Black-Scholes Formula for a European Put Option
III Implied Volatility
IV WSJ reading
V Market-Maker Risk and Delta-Hedging
VI Option Greeks
VII Gamma-Neutrality
VIII Calendar Spreads
IX Appendix: Formulas for Option Greeks

3
4
5
11
14
15
16
17
23
27
33
38
46
60
66
70
27 / 74

Implied Volatility

Volatility is unobservable

Choosing a volatility to use in pricing an option is difficult but


important

Using history of returns is not the best approach, because history is


not a reliable guide to the future.

We can invert Black-Scholes formula to obtain implied volatility

We cannot use implied volatility to assess whether an option price is


correct, but implied volatility does tell us the markets assessment of
volatility

28 / 74

Implied Volatility (contd)


I

Example: Let S = $100, K = $90, r = 8%, = 5%, and T = 1. The


market option price for a call option is $18.25. What is the volatility
that gives this option price?

We must invert the following formula


$18.25 = BSCall (100, 90, , 0.08, 1, 0.05)

We use the implied volatility function:


We find that setting = 31.73%
gives us a call price of $18.25

29 / 74

Implied Volatility (contd)


The following figure plots implied volatilities for S&P 500 options,
10/28/2004. Option prices (ask) from CBOE. Assumes
S = $1, 127.44, = 1.85%, r = 2%, and expiration date 1/22/2005.
Calls
Puts
Implied volatility

0.15

0.14

1,100

1,120

1,140

Strike
30 / 74

Implied Volatility (contd)

There is a systematic pattern of implied volatility across strike prices,


called volatility skew

The volatility skew is not related to whether an option is a put or a


call, but rather to differences in the strike price and time to expiration

Explaining these patterns is a challenge for option pricing theory

31 / 74

Using Implied Volatility

Implied volatility is important for a number of reasons


I

If you need to price an option for which you cannot observe a market
price, you can use implied volatility to generate a price consistent with
the price of traded options
Implied volatility is often used as a quick way to describe the level of
option prices on a given underlying asset. Option prices are quoted
sometimes in terms of volatility, rather than as a dollar price
Volatility skew provides a measure of how well option pricing models
work

Just as stock markets provide information about stock prices and


permit trading stocks, option markets provide information about
volatility, and, in effect, permit the trading of volatility.

32 / 74

Outline
I From Binomial Trees to the Black-Scholes Option Pricing Formula
Discrete Time vs Continuous Time
The Limiting Case of the Binomial Formula
Lognormality and the Binomial Model
Black-Scholes Assumptions
Inputs in the Binomial Model and in Black-Scholes
II Black-Scholes Formula
Black-Scholes Formula for a European Call Option
Black-Scholes Formula for a European Put Option
III Implied Volatility
IV WSJ reading
V Market-Maker Risk and Delta-Hedging
VI Option Greeks
VII Gamma-Neutrality
VIII Calendar Spreads
IX Appendix: Formulas for Option Greeks

3
4
5
11
14
15
16
17
23
27
33
38
46
60
66
70
33 / 74

Four Common Strategies in Unsettled Markets (Wall


Street Journal, October 31, 2012

34 / 74

Four Common Strategies in Unsettled Markets (Wall


Street Journal, October 31, 2012 (contd)

35 / 74

Four Common Strategies in Unsettled Markets (Wall


Street Journal, October 31, 2012 (contd)

36 / 74

Four Common Strategies in Unsettled Markets (Wall


Street Journal, October 31, 2012 (contd)

37 / 74

Outline
I From Binomial Trees to the Black-Scholes Option Pricing Formula
Discrete Time vs Continuous Time
The Limiting Case of the Binomial Formula
Lognormality and the Binomial Model
Black-Scholes Assumptions
Inputs in the Binomial Model and in Black-Scholes
II Black-Scholes Formula
Black-Scholes Formula for a European Call Option
Black-Scholes Formula for a European Put Option
III Implied Volatility
IV WSJ reading
V Market-Maker Risk and Delta-Hedging
VI Option Greeks
VII Gamma-Neutrality
VIII Calendar Spreads
IX Appendix: Formulas for Option Greeks

3
4
5
11
14
15
16
17
23
27
33
38
46
60
66
70
38 / 74

Market-Maker Risk

A market-maker stands ready to sell to buyers and to buy from


sellers.

Without hedging, an active market-maker will have an arbitrary


position generated by fulfilling customer orders. This arbitrary
portfolio has uncontrolled risk.

Consequently, market-makers attempt to hedge the risk of their


position.

We will se here how they do so.

39 / 74

Market-Maker Risk (contd)


I

Suppose a customer wishes to buy 100 European call options with


maturity of 91 days. The market-maker fills this order by selling 100
call options. To be specific, suppose that S = $41, K = $40, = 0.3,
r = 0.08 (continuously compounded), and = 0. We will let T
denote the expiration time of the option and t the present, so time to
expiration is T t. Thus, T t = 91/365 = 0.249.

Suppose that the market-maker does not hedge the written option
(naked position) and the stock has the following evolution over the
next 5 days:
Day
0
1
2
3
4
5

Stock ($)
41
42.5
39.5
37
40
40
40 / 74

Market-Maker Risk (contd)


I

We can measure the profit of the market-maker by


marking-to-market the position: if we liquidated the position
today, what would be the gain or loss?
Day
0
1
2
3
4
5

Stock ($)
41
42.5
39.5
37
40
40

Call
Position ($)
-339.47
-441.04
-246.31
-129.49
-271.04
-269.27

Daily
Profit ($)
-101.57
194.73
116.82
-141.55
1.77

41 / 74

Market-Maker Risk (contd)


Naked Position

Daily Profit

200
100
0
100
1

3
Day

42 / 74

Delta-Hedging
I

Suppose the market-maker hedges the position with shares. At time


0, the delta of a call at a stock price of $41 is 0.645.

This suggests that a $1 increase in the stock price should increase the
value of the option by approximatively $0.645.

The market-maker takes an offsetting position in shares, position that


hedges the fluctuations in the option price. We say that such a
position is delta-hedged.

Then, the market-maker rebalances the portfolio each day, by


computing the new delta of the call.

The following table summarizes delta, the number of purchased


shares, the net investment, and profit for each day for 5 days (interest
expenses are ignored for simplicity).

43 / 74

Delta-Hedging (contd)

Day

Stock
($)

Option
Delta

0
1
2
3
4
5

41
42.5
39.5
37
40
40

0.645
0.730
0.548
0.373
0.581
0.580

Stock
Position
(# shares)
64.54
73.03
54.81
37.27
58.06
58.01

Daily
Profit
(Call)

Daily
profit
(Shares)

Daily
profit
(Total)

-101.57
194.73
116.82
-141.55
1.77

96.81
-219.10
-137.02
111.82
0.00

-4.77
-24.38
-20.20
-29.73
1.77

44 / 74

Delta-Hedging (contd)
Naked Position
Delta Hedged

Daily Profit

200
100
0
100
1

3
Day

Delta hedging prevents the position from reacting to small changes in


the underlying stock. For large changes, we need to take into
account the fluctuations in delta.
45 / 74

Outline
I From Binomial Trees to the Black-Scholes Option Pricing Formula
Discrete Time vs Continuous Time
The Limiting Case of the Binomial Formula
Lognormality and the Binomial Model
Black-Scholes Assumptions
Inputs in the Binomial Model and in Black-Scholes
II Black-Scholes Formula
Black-Scholes Formula for a European Call Option
Black-Scholes Formula for a European Put Option
III Implied Volatility
IV WSJ reading
V Market-Maker Risk and Delta-Hedging
VI Option Greeks
VII Gamma-Neutrality
VIII Calendar Spreads
IX Appendix: Formulas for Option Greeks

3
4
5
11
14
15
16
17
23
27
33
38
46
60
66
70
46 / 74

Option Greeks

Option Greeks are formulas that express the change in the option
price when an input to the formula changes, taking as fixed all
other inputs.
They are used to assess risk exposures. For example:
I

A market-making bank with a portfolio of options would want to


understand its exposure to stock price changes, interest rates, volatility,
maturity, etc.
A portfolio manager wants to know what happens to the value of a
portfolio of stock index options if there is a change in the level of the
stock index.
An options investor would like to know how interest rate changes and
volatility changes affect profit and loss.

47 / 74

Option Greeks (contd)


I

Before providing detailed definition of the Greeks, lets have some


intuition on how changes in inputs affect option prices:
Change in input
St

T t (t )
r

Change in call price


Ct
Ct
Ct generally
Ct
Ct

Change in put price


Pt
Pt
Pt ambiguous
Pt
Pt

Table 1 : Changes in Black-Scholes inputs and their effect on option prices.


I

An increase in the stock price (St ) raises the chance that the call will
be exercised, thus raises the call option price. Conversely, it lowers
the put option price.

48 / 74

Option Greeks (contd)


I

An increase in volatility raises the price of a call or put option,


because it increases the expected value if the option is exercised.

Options generallybut not alwaysbecome less valuable as time to


expiration decreases, i.e., there is a time decay. There are
exceptions, for example deep-in-the-money call options on an asset
with high dividend yield and deep-in-the-money puts.

A higher interest rate reduces the present value of the strike (to be
paid by a call option holder), and thus increases the call price. The
put option entitles the owner to receive the strike, whose present
value is lower with a higher interest rate. Thus, a higher interest rate
decreases the put price.

A call entitle the holder to receive stock, but without dividends prior
to expiration. Thus, the greater the dividend yield, the lower the call
price. Conversely, a put option is more valuable when the dividend
yield is greater.
49 / 74

Option Greeks (contd)


I

The Greeks are tools that let us to quantify these relationships:


Input

Greek

Definition

Mnemonic

St

(Delta)

St

(Gamma)

St

(Elasticity)

Vega

(theta)

vega
volatility
theta
time

(rho)

(Psi)

Measures the option price change when


the stock price increases by $1
Measures the change in when the
stock price increases by $1
Measures the percentage change in the
option price when the stock price increases by 1%
Measures the option price change when
there is an increase in volatility of 1%
Measures the option price change when
there is a decrease in the time to maturity (increase in calendar time) of 1 day
Measures the option price change when
there is an increase in the interest rate
of 1% (100 basis points)
Measures the option price change when
there is an increase in the continuous
dividend yield of 1% (100 basis points)

rho r

50 / 74

Option Greeks (contd)


I

Let us come back to Table 1 and complete it with the proper signs of
the Greeks:
Input
St

t
r

Call Option
Ct
Call > 0
Call
Call > 0
Call 1
Ct
VegaCall > 0
Ct gen. Call gen. < 0
Ct
Call > 0
Ct
Call < 0

Put Option
Pt
Put < 0
Put
Put > 0
Put 0
Pt
VegaPut > 0
Pt ambig.
Put any
Pt
Put < 0
Pt
Put > 0

51 / 74

Option Greeks: Example


I

The Greeks are mathematical derivatives of the option price formula


with respect to the inputs.

Suppose that the stock price is St = $41, the strike price is K = $40,
volatility is = 0.3, the risk-free rate is r = 0.08, the time to
expiration is T t = 1, and the dividend yield is = 0. The values for
the Greeks are
Input
St

t
r

Call Option
Call = 0.691
>0
Call = 0.029
>0
Call = 4.071
1
VegaCall = 0.144
>0
Call = 0.011
gen. < 0
Call = 0.214
>0
Call = 0.283
<0

Put Option
DeltaPut = 0.309
Put = 0.029
Put = 4.389
VegaPut = 0.144
Put = 0.003
Put = 0.156
Put = 0.127

<0
>0
0
>0
any
<0
>0
52 / 74

Delta ()
Measures the change in the option price for a $1 change in the stock price:
Call
Put

Delta

0.5
0
0.5
1
20

30

40
Stock Price

50

60

53 / 74

Gamma ()
Measures the change in delta when the stock price changes:
0.04

Call
Put

Gamma

0.03

0.02

0.01

20

30

40
Stock Price

50

60

54 / 74

Elasticity ()
Measures the percentage change in the option price relative to the
percentage change in the stock price:
Call
Put

10

Elasticity

5
0
5

20

30

40
Stock Price

50

60

55 / 74

Vega
Measures the change in the option price when volatility changes (divide by
100 for a change per percentage point):
Call
Put

0.15

Vega

0.1

0.05

20

30

40
Stock Price

50

60

56 / 74

Theta ()
Measures the change in the option price with respect to calendar time, t,
holding fixed the maturity date T . To obtain per-day theta, divide by 365.
Call
Put

Theta

0.01
0
0.01
0.01
20

30

40
Stock Price

50

60

57 / 74

Rho ()
Measures the change in the option price when the interest rate changes
(divide by 100 for a change per percentage point, or by 10,000 for a
change per basis point):
0.4

Call
Put

Rho

0.2
0
0.2
0.4
20

30

40
Stock Price

50

60
58 / 74

Psi ()
Measures the change in the option price when the continuous dividend
yield changes (divide by 100 for a change per percentage point):
Call
Put

0.2

Psi

0
0.2
0.4
0.6
20

30

40
Stock Price

50

60

59 / 74

Outline
I From Binomial Trees to the Black-Scholes Option Pricing Formula
Discrete Time vs Continuous Time
The Limiting Case of the Binomial Formula
Lognormality and the Binomial Model
Black-Scholes Assumptions
Inputs in the Binomial Model and in Black-Scholes
II Black-Scholes Formula
Black-Scholes Formula for a European Call Option
Black-Scholes Formula for a European Put Option
III Implied Volatility
IV WSJ reading
V Market-Maker Risk and Delta-Hedging
VI Option Greeks
VII Gamma-Neutrality
VIII Calendar Spreads
IX Appendix: Formulas for Option Greeks

3
4
5
11
14
15
16
17
23
27
33
38
46
60
66
70
60 / 74

Gamma-Neutrality
I

Gamma hedging is the construction of options positions that are


hedged such that the total gamma of the position is zero.

We cannot do this using just the stock, because the gamma of the
stock is zero (the delta of a stock is constant and equal to 1).

Hence, we must acquire another option in an amount that offsets the


gamma of the written call.

In addition to the 91-day call from the previous example, consider a


45-strike 120-day call.

The ratio of the gamma of the two options is


K =40,T t=91
0.0606
=
= 1.1213
K =45,T t=120 0.0540

(10)

Thus, we need to buy 1.1213 of the 45-strike options for every


40-strike option we have sold.
61 / 74

Gamma-Neutrality (contd)

The Greeks resulting form this position are in the last column of the
following table:
Price ($)
Delta ()
Gamma ()
Theta ()

40-Strike Call
3.395
0.645
0.061
-0.018

45-Strike Call
1.707
0.381
0.054
-0.014

Total Position
-1.481
-0.218
0
0.002

Since delta is -0.218, we need to buy 21.8 shares of stock to be both


delta- and gamma-hedged.

62 / 74

Gamma-Neutrality (contd)

We can compare the delta-hedged position with the delta- and


gamma-hedged position. The delta-hedged position has the problem
that large moves always cause losses.

The delta- and gamma-hedged position loses less if there is a large


move down, and can make money if the stock price increases.

This can be seen from the following figure. It compares the 1-day
holding period profit for delta-hedged position described earlier and
delta- and gamma hedged position.

63 / 74

Gamma-Neutrality (contd)
Delta-hedged
Delta- and gamma-hedged

Overnight Profit ($)

10

20

30
38

39

40 41 42
Stock Price

43

44

64 / 74

Gamma-Neutrality (contd)
Delta-gamma hedging prevents the position from reacting to large
changes in the underlying stock:
Naked Position
-hedged
- and -hedged

200
Daily Profit

100
0
100
1

3
Day

65 / 74

Outline
I From Binomial Trees to the Black-Scholes Option Pricing Formula
Discrete Time vs Continuous Time
The Limiting Case of the Binomial Formula
Lognormality and the Binomial Model
Black-Scholes Assumptions
Inputs in the Binomial Model and in Black-Scholes
II Black-Scholes Formula
Black-Scholes Formula for a European Call Option
Black-Scholes Formula for a European Put Option
III Implied Volatility
IV WSJ reading
V Market-Maker Risk and Delta-Hedging
VI Option Greeks
VII Gamma-Neutrality
VIII Calendar Spreads
IX Appendix: Formulas for Option Greeks

3
4
5
11
14
15
16
17
23
27
33
38
46
60
66
70
66 / 74

Calendar Spreads

To protect against a stock price increase when selling a call, you can
simultaneously buy a call option with the same strike and greater time
to expiration.

This purchased calendar spread exploits the fact that the written
near-to-expiration option exhibits greater time decay than the
purchased far-to-expiration option, and therefore is profitable if the
stock price does not move.

67 / 74

Calendar Spreads (contd)

Suppose you sell a 40-strike call with 91 days to expiration and buy a
40-strike call with 1 year to expiration. Assume a stock price of $40,
r = 8%, = 30%, and = 0.

The premiums are $2.78 for the 91-day call and $6.28 for the 1-year
call.

Theta is more negative for the 91-day call (-0.0173) than for the
1-year call (-0.0104). Thus, if the stock price does not change over
the course of 1 day, the position will make money since the written
option loses more value than the purchased option.

68 / 74

Calendar Spreads (contd)


The profit diagram for this position for a holding period of 91 days is
displayed below

Profit in 91 days ($)

Short call
Calendar spread

10

30

40
50
Stock price ($)

60

69 / 74

Outline
I From Binomial Trees to the Black-Scholes Option Pricing Formula
Discrete Time vs Continuous Time
The Limiting Case of the Binomial Formula
Lognormality and the Binomial Model
Black-Scholes Assumptions
Inputs in the Binomial Model and in Black-Scholes
II Black-Scholes Formula
Black-Scholes Formula for a European Call Option
Black-Scholes Formula for a European Put Option
III Implied Volatility
IV WSJ reading
V Market-Maker Risk and Delta-Hedging
VI Option Greeks
VII Gamma-Neutrality
VIII Calendar Spreads
IX Appendix: Formulas for Option Greeks

3
4
5
11
14
15
16
17
23
27
33
38
46
60
66
70
70 / 74

Appendix: Formulas for Option Greeks


I

Delta () measures the change in the option price for a $1 change in


the stock price:
C
= e (T t) N (d1 )
S
P
=
= e (T t) N (d1 )
S

Call =

(11)

Put

(12)

Gamma () measures the change in delta when the stock price


changes:
2C
e (T t) N 0 (d1 )

=
S 2
S T t
2P
= Call
=
S 2

Call =

(13)

Put

(14)

71 / 74

Appendix: Formulas for Option Greeks (contd)


I

Elasticity () measures the percentage change in the option price


relative to the percentage change in the stock price:
St Call
Ct
St Put
=
Pt

Call =

(15)

Put

(16)

Vega measures the change in the option price when volatility changes
(divide by 100 for a change per percentage point):

C
= Se (T t) N 0 (d1 ) T t

P
=
= VegaCall

VegaCall =

(17)

VegaPut

(18)

72 / 74

Appendix: Formulas for Option Greeks (contd)


I

Theta () measures the change in the option price with respect to


calendar time, t, holding fixed the maturity date T . To obtain
per-day theta, divide by 365.
C
Ke r (T t ) N 0 (d2 )

= Se (T t ) N (d1 ) rKe r (T t ) N (d2 )


t
2 T t
(19)
P
=
= Call + rKe r (T t ) Se (T t )
(20)
t

Call =

Put

Rho () measures the change in the option price when the interest
rate changes (divide by 100 for a change per percentage point, or by
10,000 for a change per basis point):
C
= (T t) Ke r (T t) N (d2 )
r
P
=
= (T t) Ke r (T t) N (d2 )
r

Call =

(21)

Put

(22)
73 / 74

Appendix: Formulas for Option Greeks (contd)

Psi () Measures the change in the option price when the continuous
dividend yield changes (divide by 100 for a change per percentage
point):
C
= (T t) Se (T t) N (d1 )

P
=
= (T t) Se (T t) N (d1 )

Call =

(23)

Put

(24)

74 / 74

Option Markets 232D


4. Forwards and Futures
Daniel Andrei

Fall 2012

1 / 62

Outline
I Pricing Forwards

II Futures Contracts

11

III Financial Forwards and Futures


Index Futures
Currency Futures

13
14
24

IV Commodity Forwards and Futures

27

V Interest Rate Forwards and Futures

41

VI Options on Futures

55

2 / 62

Outline
I Pricing Forwards

II Futures Contracts

11

III Financial Forwards and Futures


Index Futures
Currency Futures

13
14
24

IV Commodity Forwards and Futures

27

V Interest Rate Forwards and Futures

41

VI Options on Futures

55

3 / 62

Alternative ways to buy a stock


1. Outright Purchase:

2. Forward:

I Pay S0

I Pay F0,T =?

I Receive security

I Receive security

Time 0

Time T

A forward contract is an arrangement in which you both pay for the


stock and receive it at time T , with the time T price specified at
time 0.

What should you pay for the stock in this case?

Arbitrage ensures that there is a very close relationship between


prices and forward prices

4 / 62

Pricing a Forward Contract


I

Let S0 be the spot price of an asset at time 0, and r the continuously


compounded interest rate. Assume that dividends are continuous and
paid at a rate .

Then the forward price at a future time T must satisfy


F0,T = S0 e (r )T

(1)

Suppose that F0,T > S0 e (r )T . Then an investor can execute the


following trades at time 0 (buy low and sell high) and obtain an
arbitrage profit:
Transaction
Buy tailed position in stock (e T units)
Borrow S0 e T
Short forward
Total

Time 0
S0 e T
+S0 e T
0
0

Cash Flows
Time T (expiration)
ST
S0 e (r )T
F0,T ST
F0,T S0 e (r )T > 0
5 / 62

Pricing a Forward Contract (contd)


I

Suppose that F0,T < S0 e (r )T . Then an investor can execute the


following trades at time 0 (buy low and sell high) and obtain once
again an arbitrage profit:
Transaction
Short tailed position in stock (e T units)
Lend S0 e T
Long forward
Total

Time 0
S0 e T
S0 e T
0
0

Cash Flows
Time T (expiration)
ST
S0 e (r )T
ST F0,T
S0 e (r )T F0,T > 0

Consequently, and assuming that the non-arbitrage condition holds,


we have
F0,T = S0 e (r )T

6 / 62

Payoff Diagram for a Forward

Payoff ($) = Profit ($)

200

Long forward ST F0,T

100

100
F0,T

Short forward F0,T ST

200
800

900

1,000

1,100

1,200

ST
7 / 62

Risk-Neutral Valuation

The risk neutral technique also works for forward contracts. When the
contract is agreed to initially, its value is 0. Its payoff at maturity is
ST F0,T

Therefore
0 = S0 e T F0,T e rT
which yields F0,T = S0 e (r )T

8 / 62

Put-Call Parity and the Forward Price


I

Consider a call and a put having both the same maturity and the
same strike price.

Let the strike price be equal to the forward price with the same
maturity:
K = F0,T = S0 e (r )T

Write the put-call parity relationship:


C0 P0 = S0 e T Ke rT
= S0 e T S0 e (r )T e rT
=0

The price of a call equals the price of a put with the same maturity
when the strike price is equal to the forward price.
9 / 62

Example: Let S = $100, = 30%, r = 8%, = 5%,


T = 1, and K = $100e (0.080.05)1 = $103.05.
I

What is the Black-Scholes call price?


The Black-Scholes call price is
$11.34

What is the Black-Scholes put price?


The Black-Scholes put price is
$11.34

10 / 62

Outline
I Pricing Forwards

II Futures Contracts

11

III Financial Forwards and Futures


Index Futures
Currency Futures

13
14
24

IV Commodity Forwards and Futures

27

V Interest Rate Forwards and Futures

41

VI Options on Futures

55

11 / 62

Futures Contracts

I
I

Exchange-traded forward contracts


Typical features of futures contracts
I
I
I

Matches buy and sell orders


Keeps track of members obligations and payments
After matching the trades, becomes counterparty

Differences from forward contracts


I
I
I

Settled daily through the mark-to-market process low credit risk


Highly liquid easier to offset an existing position
Highly standardized structure harder to customize

12 / 62

Outline
I Pricing Forwards

II Futures Contracts

11

III Financial Forwards and Futures


Index Futures
Currency Futures

13
14
24

IV Commodity Forwards and Futures

27

V Interest Rate Forwards and Futures

41

VI Options on Futures

55

13 / 62

Index Futures

Figure 1 : Listing of various index futures contracts from the Wall Street
Journal, October 6-7, 2012.

14 / 62

The S&P 500 Futures Contract


Specifications for the S&P 500 index futures contract
I

Underlying: S&P 500 index

Where traded: Chicago Mercantile Exchange

Size: $250 S&P 500 index

Months: Mar, Jun, Sep, Dec

Trading ends: Business day prior to determination of settlement price

Settlement: Cash-settled, based upon opening price of S&P 500 on


third Friday of expiration month

Suppose the futures price is 1100 and you wish to enter into 8 long
futures contracts.

The notional value of 8 contracts is


8 $250 1100 = $2, 000 1100 = $2.2 million
15 / 62

The S&P 500 Futures Contract (contd)


I

Suppose that there is 10% margin and weekly settlement (in practice
settlement is daily). The margin on futures contracts with a notional
value of $2.2 million is $220,000.

The margin balance today from long position in 8 S&P 500 futures
contracts is
Week
0

Multiplier ($)
2000.00

Futures Price
1100.00

Price Change

Margin Balance ($)


220,000.00

Over the first week, the futures price drops 72.01 points to 1027.99.
On a mark-to-market basis, we have lost
$2, 000 (72.01) = $144, 020

Thus, if the continuously compounded interest rate is 6%, our margin


balance after one week is
$220, 000 e 0.061/52 $144, 020 = $76, 233.99
16 / 62

The S&P 500 Futures Contract (contd)


I

Because we have a 10% margin, a 6.5% decline in the futures price


results in a 65% decline in margin. The margin balance after the first
week is
Week
0
1

Multiplier ($)
2000.00
2000.00

Futures Price
1100.00
1027.99

Price Change

-72.01

Margin Balance ($)


220,000.00
76,233.99

The decline in margin balance means the broker has significantly less
protection should we default. For this reason, participants are required
to maintain the margin at a minimum level, called the maintenance
margin. This is often set at 70% to 80% of the initial margin level.

In this example, the broker would make a margin call, requesting


additional margin.

We can go on for a period of 10 weeks, assuming weekly


marking-to-market and a continuously compounded risk-free rate of
6%.
17 / 62

The S&P 500 Futures Contract (contd)


I

The margin balance after a period of 10 weeks is


Week
0
1
2
3
4
5
6
7
8
9
10

Multiplier ($)
2000.00
2000.00
2000.00
2000.00
2000.00
2000.00
2000.00
2000.00
2000.00
2000.00
2000.00

Futures Price
1100.00
1027.99
1037.88
1073.23
1048.78
1090.32
1106.94
1110.98
1024.74
1007.30
1011.65

Price Change

-72.01
9.89
35.35
-24.45
41.54
16.62
4.04
-86.24
-17.44
4.35

Margin Balance ($)


220,000.00
76,233.99
96,102.01
166,912.96
118,205.66
201,422.13
234,894.67
243,245.86
71,046.69
36,248.72
44,990.57

The 10-week profit on the position is obtained by subtracting from


the final margin balance the future value of the original margin
investment:
$44, 990.57 $220, 000 e 0.0610/52 = $177, 562.60
18 / 62

The S&P 500 Futures Contract (contd)

What if the position had been forwarded rather than a futures


position, but with prices the same? In that case, after 10 weeks our
profit would have been
(1011.65 1100) $2, 000 = $176, 700

The futures and forward profits differ because of the interest earned
on the mark-to-market proceeds (in the present cases, we have
founded losses as they occurred and not at expiration, which explains
the loss).

19 / 62

Quanto Index Contracts


Specifications for the Nikkei 225 index futures contract
I

Underlying: Nikkei 225 stock index

Where traded: Chicago Mercantile Exchange

Size: $5 Nikkei 225 index

Months: Mar, Jun, Sep, Dec

Trading ends: Business day prior to determination of settlement price

Settlement: Cash-settled, based upon opening Osaka quotation of the


Nikkei 225 index on the second Friday of expiration month

There is one very important difference with the S&P 500 contract:
Settlement of the contract is in a different currency (dollars) than the
currency of denomination for the index (yen)

Consequently, the contract insulates investors from currency risk,


permitting them to speculate solely on whether the index rises or falls
20 / 62

Uses Of Index Futures

Why buy an index futures contract instead of synthesizing it using the


stocks in the index? Lower transaction costs

Asset allocation: switching investments among asset classes


Example: invested in the S&P 500 index and wish to temporarily
invest in bonds instead of the index. What to do?

I
I

Alternative #1: sell all 500 stocks and invest in bonds


Alternative #2: take a short forward position in S&P 500 index

21 / 62

Uses Of Index Futures (contd)

Suppose that the current index price, S0 , is $100, and the effective
1-year risk-free rate is 10%. The forward price is therefore $110. The
effect of owning the stock and selling forward is
Transaction
Own stock @ $100
Short forward @ $110
Total

Today
-$100
0
-$100

Cash Flows
1 year, S1 = $80 1 year, S1 = $130
$80
$130
$110 - $80
$110 - $130
$110
$110

22 / 62

Uses Of Index Futures (contd)

I
I

General asset allocation: futures overlay, alpha-porting


Cross-hedging: hedge portfolios that are not exactly the index
I
I

Cross-hedging with perfect correlation


Cross-hedging with imperfect correlation

Risk management for stock-pickers

23 / 62

Currency Futures

Figure 2 : Listing of various currency futures contracts from the Wall Street
Journal, October 6-7, 2012.
24 / 62

Currency Contracts
I

Currency forwards and futures are widely used to manage foreign


exchange risk

Suppose that T years from today you want to buy U1 with dollars.
Denote the yen-denominated interest rate by ry , the
dollar-denominated interest rate by r , and the exchange rate today
($/U) by x0

The forward price for a yen is


F0,T = x0 e (r ry )T

(2)

The forward currency rate will exceed the current exchange rate when
the domestic risk-free rate is higher than the foreign risk-free rate

Notice that equation (2) is just like equation (1), for stock index
futures, with the foreign interest rate equal to the dividend yield.
25 / 62

Currency Contracts (contd)


I

We can prove equation (2) by absence of arbitrage and by building a


synthetic currency forward: borrowing in one currency and lending in
another, which creates the same cash-flow as the forward contract:
Cash Flows
Transaction
Borrow x0 e ry T dollar at r
Convert to yen @ x0
Invest in yen-denominated bill at ry
Total

Year 0
$
+x0 e ry T
x0 e ry T

+e ry T
e ry T
0

Year T
$
x0 e (r ry )T

x0 e (r ry )T

1
1

Example: Suppose that the yen-denominated interest rate is ry = 2%


and the dollar-denominated rate is r = 6%. The current exchange
rate rate is x0 = 0.009 dollars per yen. The 1-year forward rate is
0.009e 0.060.02 = 0.009367

26 / 62

Outline
I Pricing Forwards

II Futures Contracts

11

III Financial Forwards and Futures


Index Futures
Currency Futures

13
14
24

IV Commodity Forwards and Futures

27

V Interest Rate Forwards and Futures

41

VI Options on Futures

55

27 / 62

Commodity Forwards and Futures

Figure 3 : Listing of various currency futures contracts from the Wall Street
Journal, October 6-7, 2012.
28 / 62

Introduction to Commodity Forwards and Futures


I

Financial forward prices are described by the general formula


F0,T = S0 e (r )T

At a general level, commodity forward prices can be described by the


same formula. There are, however, important differences:
I

For financial assets is the dividend yield, whereas for commodities is


the commodity lease rate
While the dividend yield for a financial asset can typically be observed
directly, the lease rate for a commodity can be estimated only by
observing the forward price

The formula for a commodity forward price is


F0,T = S0 e (r l )T

(3)

29 / 62

Forward Prices and The Lease Rate

When we observe the forward price, we can infer the lease rate.
Specifically, if the forward price is F0,T , the annualized lease rate is
F0,T
1
l = r ln
T
S0


(4)

If instead we use an effective annual interest rate, the effective annual


lease rate is
l =

1+r
(F0,T /S0 )1/T

(5)

30 / 62

The Forward Curve


I

Commodities are complex because every commodity market differs in


the details. For example:
I
I

Storage is not possible for electricity


Gold is durable and relatively inexpensive to store (compared to its
value)
Some commodities feature seasonality in production (for example, corn
in the United States is harvested primarily in the fall)

One way to observe and understand this heterogeneity is to build the


forward curve (or the forward strip), i.e., the set of prices for
different expiration dates for a given commodity
I

If on a given date the forward curve is upward-sloping, then the market


is in contango.
If the forward curve is downward sloping, the market is in
backwardation
Forward curves can have portions in backwardation and portions in
contango

31 / 62

Forward Curves for Various Commodities, May 5, 2004


Corn Futures Price (cents per bushel)

Gasoline Futures Price (cents per gallon)


130

319
125
318
Backwardation

120
317

Backwardation
Contango

115

316

110

315
1

3
4
Months to Maturity

4
5
6
Months to Maturity

Gold Futures Price ($ per ounce)

Crude Oil Futures Price ($ per barrel)


39.5

395.5
39
395
38.5
Contango

Backwardation

394.5

38
394

37.5

393.5
2

4
5
6
Months to Maturity

3
4
5
Months to Maturity

32 / 62

Futures Prices for Corn for the first Wednesday in June,


1995-2004
Futures Prices (cents per bushel)

450
400
350
300
250
200
Jul Sep

Dec

Mar May Jul Sep

Dec

Expiration Month
33 / 62

Oil Prices: Contango vs. Backwardation


I

NPR news, Dec 17, 2008: Contango in Oil Markets Explained


I

The supply glut in the oil market has led to a contango price structure,
in which oil futures are priced higher than their spot price... Usually,
the market goes into contango when there is a supply glut, when
theres too much oil on the market. And this is what weve seen
happen just in the last few months as the recession fully set in and
people realized the days of easy credit and high oil prices are gone...
Theres just a lot of oil floating around with no demand to take it up.

Wall Street Journal, Oct 28, 2011: Oil Futures Return to


Backwardation
I

Crude oil traded on the New York Mercantile Exchange has been
trading in contango, a commodity price structure in which futures
contracts for near-term delivery are cheaper than further-out contracts.
But thats changed in the last few days... Currently, we do have rising
production from Libya, which does suggest more generous supply in the
months ahead... Aside from that, were also at a time of year when
global demand is strong seasonally and low seasonal demand in the
second quarter points to a potential surplus at that timea reason for
prices to be softer then.
34 / 62

Nymex Crude Falls as Brent Divergence Widens


From the Wall Street Journal,
October 8, 2012:
I Theyre really in two different

universes... In terms of supply and


demand, theyre just two different
markets.
I Declining output in the North Sea,
tension in the Middle East, and the
Iranian oil embargo are having a
disproportionate impact on the Brent
market.
I In the U.S., the Energy Information
Administration reported domestic
crude-oil output rose to its highest
level since 1996 during the prior
week. The surge in production has
been fueled by rising flows from U.S.
shale-oil fields.
35 / 62

Gold Futures

Specifications for the NYMEX gold futures contract


I

Underlying: Refined gold bearing approved refiner stamp

Where traded: New York Mercantile Exchange

Size: 100 troy ounces

Months: Feb, Apr, Aug, Oct, out two years. Jun, Dec, out 5 years

Trading ends: Third-to-last business day of maturity month

Delivery: Any business day of the delivery month

36 / 62

Suppose that on June 6, 2001, the gold spot price is


$265.7 and the gold future price with maturity in
December is $269. The June to December interest rate
(annualized, effective) is 3.9917%. What is the annualized
6-month gold lease rate?

Using equation (5), the annualized 6-month lease rate is


6-month lease rate =

1 + 0.039917
(269/265.7)1/0.5

1 = 1.456%

37 / 62

Gold Lease Rate Slides to Lowest on Record as European


Banks Seek Dollars

Bloomberg, December 8, 2011:


I

The interest rate for lending gold in exchange for dollars plunged to the
lowest on record this week as European banks sought ways to secure
the U.S. currency amid the regions debt crisis
European banks especially are having liquidity funding problems, which
does see a lot of lending of gold and thats putting downward pressure
on lease rates

38 / 62

Coffee Standoff Tests Growers Grit

Source: Wall Street Journal, October 8, 2012


39 / 62

Gas Futures Tumble After Power Outages Sap Demand


From the Wall Street Journal, October 31, 2012:
I

Natural-gas futures tumbled 3% on Tuesday as traders bet on falling


demand from gas-fired power plants after Storm Sandy cut power to
millions of people along the East Coast.

Were faced with the loss of demand in a well-supplied market, said


Teri Viswanath, an energy analyst at BNP Paribas.

Futures prices could see further declines if data later this week and
next show falling demand and rising stockpiles.

Natural gas for December delivery settled 11.2 cents lower at $3.691
per million British thermal units on New York Mercantile Exchange
electronic trading.

After tumbling to decade lows in April under $2/MMBtu, gas futures


have made a steady climb amid rising demand from utilities. Many
power producers switched to burning gas from coal to take advantage
of low gas prices.
40 / 62

Outline
I Pricing Forwards

II Futures Contracts

11

III Financial Forwards and Futures


Index Futures
Currency Futures

13
14
24

IV Commodity Forwards and Futures

27

V Interest Rate Forwards and Futures

41

VI Options on Futures

55

41 / 62

Bond Basics
1
[1+r (0,t2 )]t2

P(0, t2 )
P(0, t1 )
0

1
[1+r (0,t1 )]t1

$1
t1 years

P0 (t1 , t2 )
I

$1

t2 years
1
[1+r0 (t1 ,t2 )]t2 t1

$1

Notation:
I
I
I
I

P(0, t1 ) and P(0, t2 ): zero coupon bond


r (0, t1 ) and r (0, t2 ): effective annual interest rate (yield to maturity)
P0 (t1 , t2 ): implied forward zero-coupon bond price (quoted at time 0)
r0 (t1 , t2 ): implied forward rate (prevailing at time 0)
42 / 62

Implied Forward Rates and Zero Coupon Bonds


I

Suppose that today is 0 and we plan to borrow $1 at t1 and pay back


at t2

We can synthetically create this by trading zero-coupon bonds


Transaction
Sell P(0, t1 )/P(0, t2 ) z-c bonds maturing @ t2
Buy 1 z-c bond maturing @ t1
Total

Time 0
+P(0, t1 )
P(0, t1 )
0

Cash Flows
Time t1 Time t2
P(0,t1 )
P(0,t
2)
$1

1)
$1 P(0,t
P(0,t2 )

It follows that the implied forward zero-coupon bond price must be


consistent with the implied forward interest rate and the zero-coupon
bond prices with maturities t1 and t2 :
P0 (t1 , t2 ) =

P(0, t2 )
1
t2 t1 =
P(0, t1 )
[1 + r0 (t1 , t2 )]

43 / 62

Forward Rate Agreements

Consider a firm expecting to borrow $100m for 91 days (3 months),


beginning 120 days from today, in June (using our previous notation,
t1 = 120/360 is the borrowing date and t2 = 211/360 is the loan
repayment date).

In June, the effective quarterly interest rate can be either 1.5% or 2%.
The implied June 91-day forward rate (the rate from June to
September) is 1.8%.

Here is the risk faced by the borrower, assuming no hedging:


Transaction

June (t1 = 120/360)

Borrow $100m

+$100m

September (t2 = 211/360)


rquarterly = 1.5% rquarterly = 2%
-$101.5m
-$102.0m

44 / 62

Forward Rate Agreements (contd)


I

A forward rate agreement (FRA) is an over-the-counter contract


that guarantees a borrowing or lending rate on a given notional
principal amount.
FRAs can be settled at maturity (in arrears) or at the initiation of
the borrowing or lending transaction
I

FRA settlement in arrears:



Payment to the borrower = rquarterly rFRA notional principal

FRA settlement at the borrowing date:


Payment to the borrower =

rquarterly rFRA
notional principal
1 + rquarterly

45 / 62

Forward Rate Agreements (contd)


I

FRA settlement in arrears:


Transaction

June (t1 = 120/360)

Borrow $100m
FRA payment if rquarterly = 1.5%
FRA payment if rquarterly = 2%
Total

+$100m

+$100m

September (t2 = 211/360)


rquarterly = 1.5% rquarterly = 2%
-$101.5m
-$102.0m
-$0.3m

+$0.2m
-$101.8m
-$101.8m

FRA settlement at the time of borrowing:


Transaction

June (t1 = 120/360)

Borrow $100m
FRA payment if rquarterly = 1.5%
FRA payment if rquarterly = 2%
Total

+$100m
$295, 566.50
+$196, 078.43
+$100m

September (t2 = 211/360)


rquarterly = 1.5% rquarterly = 2%
-$101.5m
-$102.0m
-$0.3m
+$0.2m
-$101.8m
-$101.8m

46 / 62

Eurodollar Futures
Specifications for the Eurodollar futures contract
I

Where traded: Chicago Mercantile Exchange

Size: 3-month Eurodollar time deposit, $1 million principal

Months: Mar, Jun, Sep, Dec, out 10 years, plus 2 serial months and
spot month

Trading ends: 5 A.M. (11 A.M. London) on the second London bank
business day immediately preceding the third Wednesday of the
current month

Delivery: Cash settlement

Settlement: 100 - British Bankers Association Futures Interest


Settlement Rate for 3-Month Eurodollar Interbank Time Deposits.
(This is a 3-month rate annualized by multiplying by 360/90.)

47 / 62

Eurodollar Futures (contd)

Figure 4 : Listing for the 3-month Eurodollar futures contract from the Wall
Street Journal, October 17, 2012.

48 / 62

Eurodollar Futures (contd)

Since the notional principal is $1 million, a change in annualized


LIBOR of 1 basis point would change the borrowing cost by
0.0001
$1million = $25
4

The payoff at expiration of a long Eurodollar futures contract is


[(100 rLIBOR ) Futures Price] 100 $25
Convert the change in the
futures price to basis points
Change in borrowing cost
per basis point
49 / 62

Eurodollar Futures: Example

Lets consider again the example in which we wish to guarantee a


borrowing rate for a $100m loan from June to September

Suppose the June Eurodollar futures price is 92.8. Implied 3-month


LIBOR is
100 92.8
= 1.8% over 3 months
4

As before, consider 2 possible 3-month borrowing rates in June:


I
I

1.5%, which correspond to a Eurodollar futures price in June of 94


2%, which correspond to a Eurodollar futures price in June of 92

50 / 62

Eurodollar Futures: Example (contd)

We expect to borrow $100m for 91 days, beginning 120 days from


today. Thus, we are short the interest rate.

We wish to hedge this exposure we short 100 Eurodollar futures


contracts

The futures contract settles in June:


[(92.8 94) 100 $25] 100
Transaction

June (t1 = 120/360)

Borrow $100m
Eurodollar payment if rquarterly = 1.5%
Eurodollar payment if rquarterly = 2%

+$100m
$300, 000
+$200, 000

September (t2 = 211/360)


rquarterly = 1.5% rquarterly = 2%
-$101.5m
-$102.0m

[(92.8 92) 100 $25] 100

51 / 62

Eurodollar Futures: Example (contd)


I

This is like the payment on an FRA paid in arrears, except that the
futures contract settles in June, but our interest expense is not paid
until September.

We need to tail the position in order to earn or pay interest on our


Eurodollar gain or loss before we actually have to make the interest
payment:
Number of Eurodolar contracts =

100
= 98.2318
1 + 0.018

Transaction

June (t1 = 120/360)

Borrow $100m
Eurodollar payment if rquarterly = 1.5%
Eurodollar payment if rquarterly = 2%
Total

+$100m
$294, 695
+$196, 464
+$100m

September (t2 = 211/360)


rquarterly = 1.5% rquarterly = 2%
-$101.5m
-$102.0m
-$299,116
+$200,393
-$101.799m
-$101.799m

52 / 62

Convexity Bias
I
I

The net borrowing cost appears to be a little less than 1.8%


This is not due to rounding error. Actually, the settlement structure
of the Eurodollar contract works systematically in favor of the
borrower:
I

When the interest rate turns out to be high (2%), the short Eurodollar
contract has a positive payoff and the proceeds can be reinvested at
the high realized rate
When the interest rate turns out to be low (1.5%), the short Eurodollar
contract has a negative payoff and we can found this loss until the loan
payment date by borrowing at a low rate

In order for the futures price to be fair to both the borrower and
lender, the rate implicit in the Eurodollar futures price must be higher
than a comparable FRA rate.

This difference between the FRA rate and the Eurodollar rate is called
convexity bias.
53 / 62

Suppose that on June 6, 2001, the gold spot price is


$265.7 and the gold future price with maturity in
December is $269. The June and September Eurodollar
futures prices on this date are 96.09 and 96.13. What is
the annualized 6-month gold lease rate?

54 / 62

Suppose that on June 6, 2001, the gold spot price is


$265.7 and the gold future price with maturity in
December is $269. The June and September Eurodollar
futures prices on this date are 96.09 and 96.13. What is
the annualized 6-month gold lease rate?
I

The 3-month LIBOR from June to September is


10096.09
91
400
90 = 0.988%. The 3-month LIBOR from September to
91
December is 10096.13
400
90 = 0.978%

The June to December interest rate is therefore


1.00988 1.00978 1 = 1.9763%, or 3.9917% annualized.

Using equation (5), the annualized 6-month gold lease rate is


6-month lease rate =

1 + 0.039917
(269/265.7)1/0.5

1 = 1.456%

54 / 62

Outline
I Pricing Forwards

II Futures Contracts

11

III Financial Forwards and Futures


Index Futures
Currency Futures

13
14
24

IV Commodity Forwards and Futures

27

V Interest Rate Forwards and Futures

41

VI Options on Futures

55

55 / 62

Options on Futures
I

The options we have considered so far, spot options, provide the


holder with the right to buy or sell a certain asset by a certain date
for a certain price

For futures options, the exercise of the option gives the holder a
position in the futures contract.
Why trade option on futures rather than options on the underlying
asset?

Futures contracts are, in many circumstances, more liquid than the


underlying asset
A futures price is known immediately from trading on the futures
exchange, whereas the spot price of the underlying may not be so
readily available
Futures on commodities are often easier to trade than the commodities
themselves
Futures options entail lower transactions costs than spot options in
many situations
56 / 62

Options on Futures: Mechanics

Call Futures Options (American Style)


I

When a call futures option is exercised, the holder acquires


1. A long position in the futures
2. A cash amount equal to the excess of the most recent settlement
futures price over the strike price

Put Futures Options (American Style)


I

When a put futures option is exercised, the holder acquires


1. A short position in the futures
2. A cash amount equal to the excess of the strike price over the most
recent settlement futures price

57 / 62

Options on Futures: Mechanics (contd)


I

If we compare how the spot and futures price evolve on the tree, we
observe a different solution for u and d in the case of future prices
The solution is exactly what we would get for an option on a stock
index if , the dividend yield, were equal to the risk-free rate:
St+h = St e (r )h+
Ft+h,T = Ft,T e

St
Ft,T = St e (r )nh

St+h = St e (r )h
Ft+h,T = Ft,T e

Thus, the nodes are constructed as


u = e

d = e

58 / 62

Options on Futures Contracts: Example

An option has a gold futures contract as the underlying asset.


Assume S = $300, K = $300, = 0.1, r = 0.05, T = 1 year, = 0.02,
and h = 1/3.

The following figure shows the tree for pricing an American call
option on a gold futures contract.

59 / 62

Options on Futures Contracts: Example (contd)


S3uuu = F3uuu = $367.6
F2uu = $347.0
S2uu = $343.5
F1u = $327.5
F0 = $309.1

S1u = $321.0

S3uud = F3uud = $327.5


F2ud = $309.1
S2ud = $306.1

S0 = $300.0
F1d = $291.8
S1d = $286.0

S3udd = F3udd = $291.8


F2dd = $272.7
S2dd = $275.4

S3ddd = F3ddd = $260.0

60 / 62

Options on Futures Contracts: Example (contd)


I

Binomial tree for pricing an


American call option on a gold
futures contract
F1u = $327.5

F0 = $309.1
C0 = $17.1
C0,NO = $17.1
C0,EX = $9.1

C1u
u
C1,NO
u
C1,EX
F1d
C1d
d
C1,NO

= $29.1
= $29.1
= $27.5

F3uuu = $367.6
F2uu = $347.0
C2uu
uu
C2,NO
uu
C2,EX

= $47.0
= $46.2
= $47.0

F2ud = $309.1
= $13.1

= $291.8

C2ud
ud
C2,NO

= $6.3

ud
C2,EX

= $9.1

= $6.3

d
C1,EX
= $0

C3uuu = $67.6

F3uud = $327.5
C3uud = $27.5

= $13.1

F2dd = $275.4

F3udd = $291.8
C3udd = $0

C2dd = $0
dd
C2,NO
= $0
dd
C2,EX
= $0

C,NO = value of call if not exercised


C,EX = value of call if exercised

F3ddd = $260.0
C3ddd = $0

61 / 62

Options on Futures Contracts: Example (contd)

Early exercise is optimal when the futures price is $347.0 (which


corresponds to a $343.5 spot price for the gold).

The intuition for early exercise is that when an option on a futures


contract is exercised, the option holder pays nothing, is entered into a
futures contract, and receives mark-to-market proceeds of the
difference between the strike price and the futures price.

The motive for exercise is the ability to earn interest on the


mark-to-market proceeds.

62 / 62

Option Markets 232D


5. Other Topics

Daniel Andrei

Fall 2012

1 / 67

Outline
I Swaps
Commodity Swaps
Interest Rate Swaps
Variance and Volatility Swaps

3
5
11
16

II Measurement and Behavior of Volatility

26

III Extending the Black-Scholes Model


Option Pricing When the Stock Price Can Jump
Stochastic Volatility

35
37
42

IV Revision

49

V About My Current Research

60

2 / 67

Outline
I Swaps
Commodity Swaps
Interest Rate Swaps
Variance and Volatility Swaps

3
5
11
16

II Measurement and Behavior of Volatility

26

III Extending the Black-Scholes Model


Option Pricing When the Stock Price Can Jump
Stochastic Volatility

35
37
42

IV Revision

49

V About My Current Research

60

3 / 67

Introduction to Swaps

A swap is a contract calling for an exchange of payments, on one or


more dates, determined by the difference in two prices

A swap provides a means to hedge a stream of risky payments

A single-payment swap is the same thing as a cash-settled forward


contract

4 / 67

An Example of a Commodity Swap

An industrial producer, IP Inc., needs to buy 100,000 barrels of oil 1


year from today and 2 years from today

The forward prices for delivery in 1 year and 2 years are $20 and
$21/barrel

The 1- and 2-year zero-coupon bond yields are 6% and 6.5%


(annualized, effective)

5 / 67

An Example of a Commodity Swap (contd)


I

IP can guarantee the cost of buying oil for the next 2 years by
entering into long forward contracts for 100,000 barrels in each of the
next 2 years. The present value of this cost per barrel is
$21
$20
+
= $37.383
1.06 1.0652

Thus, IP could pay an oil supplier any payment stream with a present
value of $37.383. Typically, a swap will call for equal payments in
each year

For example, the payment per year per barrel, x , will have to satisfy
the following equation
x
x
+
= $37.383
1.06 1.0652

We find x = $20.483 and we say that the 2-year swap price is $20.483
6 / 67

Computing the Commodity Swap Rate

Suppose there are n swap settlements, occurring on dates ti ,


i = 1, ..., n

The price of a zero-coupon bond maturing on date ti is P(0, ti )

The fixed payment on a commodity swap is


F =

Pn
i=1 P(0, ti )F0,ti
P
n
i=1 P(0, ti )

(1)

where F0,ti is the forward price with maturity ti


I

The commodity swap price is a weighted average of commodity


forward prices, where zero-coupon bond prices are used to determine
the weights

7 / 67

Computing the Commodity Swap Rate (contd)


I

A buyer with seasonally varying demand (e.g., someone buying gas for
heating) might enter into a swap in which quantities vary over time

The swap price with seasonally-varying quantities is


F =

Pn
i=1 Qti P(0, ti )F0,ti
P
n
i=1 Qti P(0, ti )

(2)

where Qti is the quantity of gas purchased at time ti


I

when Qt = 1, the formula is the same as equation (1), when the


quantity is not varying

It is also possible for prices to be time-varying (e.g., a gas buyer who


needs gas for heating can enter into a swap in which the summer
price is fixed at a low value, and then winter price is then determined
by the zero present value condition)

8 / 67

Physical Versus Financial Settlement


$20.483
Oil Buyer

Oil

Swap Counterparty

Physical settlement
Spot $20.483

Oil Buyer

Swap Counterparty

Spot
Oil

Oil Seller

Financial settlement

9 / 67

Swaps are nothing more than forward contracts coupled


with borrowing and lending money
I

Consider the swap price of $20.483/barrel. Relative to the forward


curve price of $20 in 1 year and $21 in 2 years, we are overpaying by
$0.483 in the first year, and we are underpaying by $0.517 in the
second year

Thus, by entering into the swap, we are lending the counterparty


money for 1 year. The interest rate on this loan is
0.517
1 = 7%
0.483

Given 1- and 2- year zero-coupon bond yields of 6% and 6.5%, the


1-year implied forward yield from year 1 to year 2 is
r0 (1, 2) =

P(0, 1)
1 = 7%
P(0, 2)
10 / 67

Interest Rate Swaps

The notional principal of the swap is the amount on which the


interest payments are based

The life of the swap is the swap term or swap tenor

If swap payments are made at the end of the period (when interest is
due), the swap is said to be settled in arrears

11 / 67

An Example of an Interest Rate Swap

XYZ Corp. has $200M of floating-rate debt at LIBOR, i.e., every year
it pays that years current LIBOR

XYZ would prefer to have fixed-rate debt with 3 years to maturity

XYZ could enter a swap, in which they receive a floating rate and pay
the fixed rate, which is 6.9548%

12 / 67

An Example of an Interest Rate Swap (contd)

Pay LIBOR
Lender
Borrower (XYZ)

Pay 6.9548%
Receive LIBOR

Swap Counterparty

On net, XYZ pays 6.9548%


XYZ net payment = LIBOR + LIBOR 6.9548% = 6.9548%

13 / 67

Computing the Swap Rate


I

Suppose there are n swap settlements, occurring on dates ti ,


i = 1, ..., n

The implied forward interest rate from date ti1 to date ti , known at
date 0, is r0 (ti1 , ti )

The price of a zero-coupon bond maturing on date ti is P(0, ti )

The fixed swap rate, R, is


Pn

R=

i=1 P(0, ti )r (ti1 , ti )


Pn
i=1 P(0, ti )

(3)

where ni=1 P(0, ti )r (ti1 , ti ) is the present value of interest payments


P
implied by the strip of forward rates, and ni=1 P(0, ti ) is the present
value of a $1 annuity when interest rates vary over time
P

14 / 67

Computing the Swap Rate (contd)

We can rewrite equation (3) to make it easier to interpret


R=

n
X
i=1

"

P(0, ti )
Pn
r (ti1 , ti )
j=1 P(0, tj )

(4)

Thus, the fixed swap rate is a weighted average of the implied forward
rates, where zero-coupon bond prices are used to determine the
weights

15 / 67

Variance and Volatility Swaps: Why Trade Volatility?

Just as stock investors think they know something about the direction
of the stock market, or bond investors think they can foresee the
probable direction of interest rates, so you may think you have insight
into the level of future volatility

What do you do if you simply want exposure to a stocks volatility?

Stock options are impure: they provide exposure to both the direction
of the stock price and its volatility

The easy way to trade volatility is to use volatility swaps, sometimes


called realized volatility forward contracts

These products provide pure exposure to volatility (and only to


volatility)

16 / 67

Volatility Swaps

A stock volatility swap is a forward contract on annualized volatility.


Its payoff at expiration is equal to
(R Kvol ) N

(5)

where
I

R is the realized stock volatility (quoted in annual terms) over the life
of the contract
Kvol is the annualized volatility delivery price, typically quoted as a
volatility, for example 30%
N is the notional amount of the swap in dollars per annualized
volatility point, for example N = $250, 000/(volatility point)

17 / 67

Volatility Swaps (contd)


I

The holder of a volatility swap at expiration receives N dollars for


every point by which the stocks realized volatility R has exceeded
the volatility delivery price Kvol

He or she is swapping a fixed volatility Kvol for the actual (floating)


future volatility R
The procedure for calculating the realized volatility should be clearly
specified with respect to the following aspects:

I
I
I

I
I

How frequently the return is measured


Whether returns are continuously compounded or arithmetic
Whether the variance is measured by subtracting the mean or by simply
squaring the returns
The period of time over which variance is measured
How to handle days on which trading does not occur

18 / 67

Uses for Volatility Swaps

Directional trading of volatility levels. Clients who want to


speculate on the future levels of stock or index volatility can go long
or short realized volatility with a swap

Hedging implicit volatility exposure. Risk arbitrageurs, investors


following active benchmarking stragegies, portfolio managers, equity
funds are implicitly short volatility. They can hedge this exposure by
going long realized volatility with a swap

19 / 67

Variance Swaps

A variance swap is a forward contract on annualized variance, the


square of the realized volatility. Its payoff at expiration is equal to


R2 Kvar N

(6)

where
I

I
I

R2 is the realized stock variance (quoted in annual terms) over the life
of the contract
Kvar is the delivery price for variance, for example (30%)2
N is the notional amount of the swap in dollars per annualized
volatility point squared, for example N = $100, 000/(volatility point)2

20 / 67

Variance vs. Volatility Contracts


10

Variance Swap

Payoff

5
0
Kvol

5
10
Volatility Swap
15
15

20

25

30

35

40

R
21 / 67

Replicating Variance Swaps

If you own a portfolio of options of all strikes, weighted in inverse


proportion to the square of the strike level, you will obtain an
exposure to variance that is independent of stock price, just what is
needed to trade variance.

There is no simple replication strategy for synthesizing a volatility


swap.

We can approximate a volatility swap by statically holding a suitably


chosen variance contract.

22 / 67

The CBOE Volatility Index VIX

In 1993, the Chicago Board Options Exchange (CBOE) introduced


the CBOE Volatility Index, VIX, which was originally designed to
measure the markets expectation of 30-day volatility implied by
at-the-money S&P 100 Index option prices

Ten years later in 2003, CBOE and Goldman Sachs updated the VIX
to reflect a new way to measure expected volatility. The new VIX is
based on the S&P 500 Index, and estimates expected volatility by
averaging the weighted prices of puts and calls over a wide range of
strike prices.

23 / 67

The VIX Calculation


I

The CBOE utilizes a wide variety of strike prices for SPX puts and
calls to calculate the VIX
The generalized formula used in the VIX calculation is

2
2 X Ki rT
1 F0,T
2 X Ki rT
e Put(Ki ) +
e Call(Ki )
1

=
T
T
T K0
Ki2
Ki2
2

Ki K0

Ki >K0

where
I
I

I
I
I

is VIX/100
T is time to expiration (in order to arrive at a 30 day implied volatility
value, the calculation blends options expiring on two different dates,
with the result being an interpolated implied volatility number)
F0,T is the forward index price
Ki is the strike price of i th out-of-the-money option
r is the risk-free interest rate to expiration

24 / 67

Why should we care about VIX?


I

VIX provides important information about investor sentiment. Since


volatility often signifies financial turmoil, the VIX is often referrred to
as the investor fear gauge.
Investors can use VIX options and VIX futures to hedge their
portfolios. The VIX is a good hedging tool because it has a strong
negative correlation to the S&P 500

80
60
40
20
Jan 2004

Jan 2006

Jan 2008

Jan 2010

Jan 2012
25 / 67

Outline
I Swaps
Commodity Swaps
Interest Rate Swaps
Variance and Volatility Swaps

3
5
11
16

II Measurement and Behavior of Volatility

26

III Extending the Black-Scholes Model


Option Pricing When the Stock Price Can Jump
Stochastic Volatility

35
37
42

IV Revision

49

V About My Current Research

60

26 / 67

Volatility is Crucial in Finance

for forecasting return

for the pricing of derivatives

for asset allocation (trade-off between return and risk)

for risk management (evaluation of the risk of a portfolio)

27 / 67

The major problem with volatility is that it is not directly


observable from returns

Unconditional volatility is estimated as the sample standard


deviation
v
u
T
u1 X
=t
(rt )2

(7)

t=1

where rt is the log return on period t and is the sample mean over
T periods
I

However, volatility is actually not constant through time. Therefore,


conditional volatility t is a more relevant measure of risk at time t.

28 / 67

Volatility is not constant through time (volatility


clustering)
SP500 return
0.2
0.15
0.1
0.05
0
0.05
0.1
0.15
0.2
0.25

200

400

600

800

1000

1200

1400

29 / 67

Using squared daily returns to proxy volatility will


produce a very noisy volatility estimator
SP500 squared return
0.045
0.04
0.035
0.03
0.025
0.02
0.015
0.01
0.005
0

200

400

600

800

1000

1200

1400

30 / 67

An alternative measure of volatility is the absolute return


SP500 absolute return
0.25

0.2

0.15

0.1

0.05

200

400

600

800

1000

1200

1400

31 / 67

The GARCH Model


I

The ARCH (AutoRegressive Conditional Heteroskedasticity)


model has been introduced by Engle (1982)

Basic idea: unexpected returns t are serially uncorrelated but


dependant. The dependency in t is described by a quadratic function
of its lagged values:
rt = + t
t = t zt
t2 = + 1 2t1 + ... + p 2tp
zt iid N(0, 1)

Due to the large persistence in volatility, the ARCH model often


requires a large p to fit the data

32 / 67

The GARCH Model (contd)

In such cases, it is more parsimonious to use the GARCH


(Generalized ARCH) model proposed by Bollerslev (1986). The
GARCH(1,1) model is defined as
rt = + t
t = t zt
2
t2 = + 2t1 + t1

zt iid N(0, 1)

33 / 67

GARCH(1,1) volatility for the S&P 500 (weekly data)


Volatility estimated using a GARCH(1,1) model
0.8
0.7
0.6
0.5
0.4
0.3
0.2
0.1
0

200

400

600

800

1000

1200

1400

34 / 67

Outline
I Swaps
Commodity Swaps
Interest Rate Swaps
Variance and Volatility Swaps

3
5
11
16

II Measurement and Behavior of Volatility

26

III Extending the Black-Scholes Model


Option Pricing When the Stock Price Can Jump
Stochastic Volatility

35
37
42

IV Revision

49

V About My Current Research

60

35 / 67

Extensions of the Black-Scholes Model

Three extensions
1. The Merton jump diffusion model
2. The Constant Elasticity of Variance model (CEV)
3. The Heston model

36 / 67

Valuation Formula when the Underlying Can Jump

Stock prices sometimes move more than would be expected from a


lognormal distribution:
I

If market volatility is 20% and the expected return is 15%, a one-day


5% drop in the market occurs about once every 2.5 million days
A 20% drop (as in October 1987) is virtually impossible if prices are
lognormally distributed

Merton (1976) used the Poisson distribution to count the number of


price jumps that occur over a period of time.
The Poisson distribution is summarized by the parameter :
I
I

h is the probability that one event occurs over the short interval h.
Thus, is like an annualized probability of the event occurring over a
short interval.

37 / 67

Valuation Formula when the Underlying Can Jump (contd)


I

Let the jump magnitude be Y . That is, if S is the pre-jump price,


Y S is the post-jump price

Let us assume that Y is lognormally distributed (hence, this is called


the Poisson-lognormal model):


ln(Y ) N J , J2

You can interpret ln(Y ) as the continuously compounded stock return


if there is a jump; ln(Y ) can go from to +

You can interpret Y 1 as the effective stock return if there is a


jump; Y 1 can go from 1 to , or Y can go from 0 to
Thus, the Merton formula for pricing with jumps requires 3 extra
parameters:

1. The jump probability,


2. The expected size of a jump when one does occur, J
3. The volatility of the jump magnitude, J
38 / 67

Valuation Formula when the Underlying Can Jump (contd)


I

The resulting process for the stock is


(

0,
dSt
= ( k) dt + dZ +
St
Y 1,
I

if there is no jump
if there is a jump

(8)

Merton (1976) shows that with the stock following equation (8), and
with jumps diversifiable, the price of an European call is
0 T
X
e
(0 T )i
i=0

i!

BSCall S, K ,

i 2
iJ
2 J , r k +
, T , (9)
T
T

39 / 67

Jump Risk and Implied Volatility


I

Suppose that the stock can jump to zero (i.e., Y = 0) with = 0.5%
probability per year

In this case, the value of a European call becomes


BSCall (S, K , , r + , T , )

Suppose that other parameters are: S = $40, = 30%, r = 8%,


T = 0.25, and = 0. The jump parameters are: = 0.005,
J = ln(0) = , and J = 0

We do the following experiment: generate correct option pricesi.e.,


prices properly accounting for the jumpfor a variety of strikes

We then ask what implied volatility we would compute for these


options using the ordinary Black-Scholes formula

40 / 67

Jump Risk and Implied Volatility (contd)


0.34
CallJump = 10.67, CallStd = 10.63
Implied Volatility

0.33

0.32

0.31

0.3
30

35

40

45
50
Strike Price

55

60

41 / 67

Constant Elasticity of Variance (CEV) Model


I

Cox (1975) proposed the constant elasticity of variance (CEV) model

Volatility varies with the level of the stock price


2
dS
= ( )dt +
S 2 dZ
S

The instantaneous standard deviation of the stock return is


(S) =
S

I
I
I

2
2

If < 2 volatility decreases with the stock price


If > 2 volatility increases with the stock price
If = 2 the CEV model reduces to the lognormal process

42 / 67

The CEV Call Price


I

For the case < 2, the CEV call price is


Se

2
, 2x
1 Q 2y , 2 +
2



Ke

rT

2
Q 2x ,
, 2y
2


(10)

where
k=

2(r )

2 (2 )(e (r )(2)T

x = ks

y = kK
I

1)

2 (r )(2)T

Q(a, b, c) dentoes the noncentral Chi-squared distribution function


with b degrees of freedom and noncentrality parameter c, evaluated
at a
43 / 67

The CEV Call Price (contd)

For the case > 2, the CEV call price is


Se

2
, 2y
1 Q 2x ,
2



Ke

rT

2
Q 2y , 2 +
, 2x
2


When < 2, the CEV model generates a Black-Scholes implied


volatility skew

Implied volatility decreases with the option strike price

(11)

44 / 67

Implied Volatility in the CEV Model

Suppose the usual parameters: S = $40, = 30%, r = 8%, T = 0.25,


and = 0.

Additionally, = 1 and
= 0.3 40 = 1.897

Now we do the following experiment: generate correct option


pricesi.e., prices properly accounting for time-varying volatilityfor
a variety of strikes

We then ask what implied volatility we would compute for these


options using the ordinary Black-Scholes formula

45 / 67

Implied Volatility in the CEV Model (contd)


0.34

Implied Volatility

CallJump = 10.67, CallStd = 10.63


CallCEV = 10.65, CallStd = 10.63

0.32

0.3

0.28

30

35

40

45
50
Strike Price

55

60

46 / 67

The Heston Stochastic Volatility Model


I

The model allows volatility to vary stochastically but still be


correlated with the stock price
q
dS
= ( )dt + v (t)dZ1
S
q

dv (t) = k(
v v (t))dt + v

v (t)dZ2

(12)
(13)

v (t) is the volatility (instantaneous standard deviation of the stock


return)

The brownians Z1 and Z2 can be correlated:


E (dZ1 dZ2 ) = dt

Hestons stochastic volatility model has an integral solution that can


be evaluated numerically
47 / 67

Implied Volatility in the Heston Model

48 / 67

Outline
I Swaps
Commodity Swaps
Interest Rate Swaps
Variance and Volatility Swaps

3
5
11
16

II Measurement and Behavior of Volatility

26

III Extending the Black-Scholes Model


Option Pricing When the Stock Price Can Jump
Stochastic Volatility

35
37
42

IV Revision

49

V About My Current Research

60

49 / 67

Below is a profit diagram for a position. In the table below,


record option quantities which construct the diagram
100
Slope=1

Profit

50
0
50

Slope=1
100

Strike
Calls

80

120

50
100
150
Stock Price in one year

200

Shares
50 / 67

Below is a profit diagram for a position. In the table below,


record option quantities which construct the diagram
100
Slope=1

Profit

50
0
50

Slope=1
100

Strike
Puts

80

120

50
100
150
Stock Price in one year

200

Shares
51 / 67

Which of the following options will NOT be exercised


early?

(a) Put on a dividend paying stock


(b) Call on a dividend paying stock
(c) Put on a non-dividend paying stock
(d) Call on a non-dividend paying stock

52 / 67

Why is an American call option rarely exercised early, and


thus priced similar to European options?

(a) Early exercise requires purchasing the stock


(b) Most option sellers cannot deliver the stock
(c) Option prices usually exceed intrinsic values
(d) Short sellers disrupt delivery

53 / 67

Assume that an investor is currently holding a reverse


straddle position (i.e. a short put and short call), which is
currently a profitable investment. All else being equal,
what would this investor like to happen to vega?

(a) Decrease
(b) Increase
(c) Stay constant
(d) Indifferent

54 / 67

Suppose the delta of an option is 5.15

(a) It is a call option


(b) It is a put option
(c) Could be either call or put

55 / 67

The price of an option is decreasing with the stock price

(a) It is a call option


(b) It is a put option
(c) Could be either call or put

56 / 67

The price of an option is decreasing with the strike price

(a) It is a call option


(b) It is a put option
(c) Could be either call or put

57 / 67

What actions are required to both delta-hedge and


gamma-hedge a written option position?

(a) No action is required


(b) We must buy more bonds
(c) We must buy more shares of the underlying
(d) We must long a different option so as to offset the short call gamma.

58 / 67

During periods when measured volatility is high, the typical


day tends to exhibit high volatility. This behavior is
referred to as volatility ______

(a) Clustering
(b) Smile
(c) Skew
(d) Stochasticity

59 / 67

Outline
I Swaps
Commodity Swaps
Interest Rate Swaps
Variance and Volatility Swaps

3
5
11
16

II Measurement and Behavior of Volatility

26

III Extending the Black-Scholes Model


Option Pricing When the Stock Price Can Jump
Stochastic Volatility

35
37
42

IV Revision

49

V About My Current Research

60

60 / 67

February 2010: Wall Street takes break for Tiger Woods


apology

Source: Bloomberg, By Michael Patterson and Eric Martin - February 19, 2010 14:16 EST

61 / 67

We explain the strong positive relationship between


investors attention and stock market volatility
S&P500 Volatility

0.4
0.3
0.2
0.1
2004

2005

2006

2007

2008

2009

2010

2011

2009

2010

2011

Focus on Economic News

3
2
1
2004

2005

2006

2007

2008

62 / 67

Bottom fishing: The quest for the bottoming-out point


that suggests a return to more bullish days

Source: Yu (2009)

63 / 67

Disagreement over the speed of the reversion of the


economy drives volatility
0.3

0.2

Stocks Diffusion

0.1

0.1

0.2

0.3

0.4

F
10

G
20

30

40

50

Time
64 / 67

Volatility is clustered

Returns
0.06
0.04
0.02

10

20

30

40

50

Time
-0.02
-0.04
-0.06

65 / 67

May 1995: IBM secretary Lorraine Cassano was asked to


photocopy some papers...
May 1995
1 2 3 4 5 6 7
8 9 10 11 12 13 14
15 16 17 18 19 20 21
22 23 24 25 26 27 28
29 30 31
June 1995
1 2 3 4
5 6 7 8 9 10 11
12 13 14 15 16 17 18
19 20 21 22 23 24 25
26 27 28 29 30

66 / 67

May 1995: IBM secretary Lorraine Cassano was asked to


photocopy some papers...
May 1995
1 2 3 4 5 6 7
8 9 10 11 12 13 14
15 16 17 18 19 20 21
22 23 24 25 26 27 28
29 30 31
June 1995
1 2 3 4
5 6 7 8 9 10 11
12 13 14 15 16 17 18
19 20 21 22 23 24 25
26 27 28 29 30

66 / 67

May 1995: IBM secretary Lorraine Cassano was asked to


photocopy some papers...
May 1995
1 2 3 4 5 6 7
8 9 10 11 12 13 14
15 16 17 18 19 20 21
22 23 24 25 26 27 28
29 30 31
June 1995

Takeover of Lotus by IBM


(to be announced June 5)

1 2 3 4
5 6 7 8 9 10 11
12 13 14 15 16 17 18
19 20 21 22 23 24 25
26 27 28 29 30

66 / 67

May 1995: IBM secretary Lorraine Cassano was asked to


photocopy some papers...
May 1995
1 2 3 4 5 6 7
8 9 10 11 12 13 14
15 16 17 18 19 20 21
22 23 24 25 26 27 28
29 30 31
June 1995

Takeover of Lotus by IBM


(to be announced June 5)

The information
percolated during 6 hours

1 2 3 4
5 6 7 8 9 10 11
12 13 14 15 16 17 18
19 20 21 22 23 24 25
26 27 28 29 30

66 / 67

May 1995: IBM secretary Lorraine Cassano was asked to


photocopy some papers...
May 1995
1 2 3 4 5 6 7
8 9 10 11 12 13 14
15 16 17 18 19 20 21
22 23 24 25 26 27 28
29 30 31
June 1995

Takeover of Lotus by IBM


(to be announced June 5)

1 2 3 4
5 6 7 8 9 10 11
12 13 14 15 16 17 18
19 20 21 22 23 24 25
26 27 28 29 30

The information
percolated during 6 hours

Investors: 25
Investment: $5000 000
Trading profits: $10 3000 000

66 / 67

May 1995: IBM secretary Lorraine Cassano was asked to


photocopy some papers...
May 1995
1 2 3 4 5 6 7
8 9 10 11 12 13 14
15 16 17 18 19 20 21
22 23 24 25 26 27 28
29 30 31
June 1995

Takeover of Lotus by IBM


(to be announced June 5)

1 2 3 4
5 6 7 8 9 10 11
12 13 14 15 16 17 18
19 20 21 22 23 24 25
26 27 28 29 30

The information
percolated during 6 hours

Investors: 25
Investment: $5000 000
Trading profits: $10 3000 000

Source: New York Daily News, p. 5, Thursday, May 27th 1999.


U.S. SECURITIES AND EXCHANGE COMMISSION Litigation Release No. 16161, MAY 26, 1999.

66 / 67

The data are never for the whole society given to a single
mind (Hayek, 1945, p. 519)

Prices aggregate knowledge

Language aggregates knowledge

67 / 67

The data are never for the whole society given to a single
mind (Hayek, 1945, p. 519)

Prices aggregate knowledge

Language aggregates knowledge

Word-of-mouth communication generates momentum


and reversal in asset returns

Word-of-mouth communication generates persistent


volatility

67 / 67

You might also like